UPSC Placement Paper

UPSC Placement Paper NDA Previous Year Exam Paper English Language Directions (For the 10 items which follow) : Each of the following items consists of a sentence followed by four words or groups of words. Select the synonym of the word or words as per the context.(occurring in the sentence in capital letters) 1. The convict's INGENUOUS explanation brought tears in every eye. (a) Candid (b) Secret (c) Insincere (d) Consistent 2. The ENORMITY of the population problem is irksome. (a) Intensity (b) Vastness (c) Cruelty (d) Fragility 3. EXAGGERATION of facts would always lead to confusion. (a) Simplification (b) Negation (c) Emancipation (d) Amplification 4. After his father's death, he became INSOLVENT. (a) Rich (b) Poor (c) Bankrupt (d) Nonchalant 5. He INDUCES human beings to want things they don't want. (a) Influences (b) Dictates (c) Persuades (d) Appreciates 6. His information is not AUTHENTIC. (a) Real (b) Reliable (c) Believable (d) Genuine 7. He is not CUT OUT to be a teacher. (a) Trained (b) Suited (c) Selected (d) Guided 8. The Chief Minister REPRIMANDED the officers for their methods of handling the public. (a) Adulated (b) Scolded severely (c) Disapproved of (d) Suspended 9. Sin is the sole calamity that a wise man should APPREHEND. (a) Give up (b) Discourage (c) Comprehend (d) Fear 10. We must be proud of our cultural HERITAGE. (a) Things we make for ourselves (b) Things we have lost (c) Things we have from the past (d) Things we do for others Directions (For the 10 items which follow) Each of the following items consists of a sentence followed by four words. Select the antonym of the word (occurring in the sentence in capital letters) as per the context. 11. The COMPLAINANT was not supportive of providing all facts in the court. (a) Defendant (b) Advocate (c) Indulgence (d) Servant 12. Living in a SOLITARY place brings in some kind of satisfaction. (a) Limited (b) Exotic (c) Healthy (d) Populous 13. They spent a DISTURBED night after hearing the tragic news. (a) Restless (b) Sleepless (c) Mournful (d) Peaceful 14. We have CREATED a beautiful new house from out of the old ruin. (a) Destroyed (b) Built (c) Constructed (d) Planned 15. A large number of designer clothes are REVOLTING. (a) Inviting (b) Beautiful (c) Fashionable (d) Delightful 16. Though they are twins, they look very DIFFERENT. (a) Regular (b) Same (c) Similar (d) Uniform 17. RECESSION is a major cause of unemployment. (a) Education (b) Inflation (c) Poverty (d) Computerization 18. He finally CONCEDED that he was involved in smuggling. (a) Admitted (b) Accepted (c) Denied (d) Concealed 19. The accused emphatically DENIED the charge in the court. (a) Accepted (b) Agreed (c) Asserted (d) Affirmed 20. This cloth is a COARSE of touch. (a) Delicate (b) Rough (c) Painful (d) Harsh General Knowledge 1. A glass of water does not turn into ice as it reaches 0 °C. It is because (a) water does not solidify at 0 °C (b) a certain amount of heat must be supplied to the glass of water so as to solidify (c) a certain amount of heat must be taken out from the glass of water so as to solidify (d) water solidifies at 0 K. only 2. The main power supply in India is at 220 V, whereas that in the US is at 110 V. Which one among the following statements in this regard is correct? (a)110 V is safer but more expensive to maintain (b)110 V is safer and cheaper to maintain (c)110 V leads to lower power loss (d)110V works better at higher latitudes 3. The equatorial rain forest is also known as (a) Savanna (b) Campos (c) Selva (d) Lanose 4. Which of the following is not a micronutrient for a plant? (a) Iron (b) Magnesium (c) Molybdenum (d) Manganese 5. Which of the following statements is/are correct? 1. Ligaments are highly flexible. 2. Ligaments connect muscles and bones. 3. Ligaments contain very little matrix. Select the correct answer using the code given below : Code : (a) 1, 2 and 3 (b) 1 and 3 only (c) 2 and 3 only (d) 1 only 6. Cell membrane is selectively permeable because (a) it is made up of selected organic molecules (b) it does not allow transport of some substances from region of higher concentration to theregion of lower concentration (c) the movement of organic molecules occurs only at specific concentration (d) it allows the movement of certain molecules in and out of the cell while the movement of other molecules is prevented 7. Certain parts of a plant can be bent easily without breaking. This flexibility in certain parts, like leaf and stem, can be attributed to the abundance of (a) parenchyma (b) collenchyma c) sclerenchyma (d) xylem and phloem 8. The following equation is an example of a redox reaction, in which C12 is the oxidizing agent and FeBr3 is the reducing agent : 2FeBr3 (aq) + 3Cl 2 (g) = 2FeCl 3 (aq) + 3Br2 (1) Which one among the following statements is incorrect for this redox reaction? (a) Oxidizing agents are themselves reduced (b) Reducing agents gainor appear to gain electrons (c) Reducing agents are themselves oxidized (d)Oxidizing agents oxidize other substances 9. Which one among the following is the correct order of amount of time (CaO), silica (Si02 ), alumina (AI -203) and ferric oxide (Fe203) in Portland cement? (a)CaO > Si02 >A ' 203 > Fe 203 (b)Si02 > CaO > Fe 203 > A1203 (c)A1203 > Si02 > CaO > Fe 203 (d)Fe 203 > A1203 > Si02 > CaO 10. Which one among the following transitions of electron of hydrogen atom emits radiation of the shortest wavelength? ( a ) n=2 to n= 1 ( b ) n=3 to n=2 (c) n=4 to n= 3 (d) n = 5 to n = 4 11. Which one among the following is the most appropriate statement with respect to the atomic weight of an element? (a) The atomic weight of a element is the sum total of the number of protons and neutrons present in the atom of the element (b) Unlike mass number, the atomic weight of an element can be a fraction (c) The atomic weight of an element is a whole number (d) The atomic weight of all the atoms in an element is the same 12. During the discussion on the Demands for Grants, motions can be moved to reduce the amount of a demand. Such a motion is called a `cut-motion'. Which of the following categories are classified as 'cut-motion? 1. Disapproval of policy cut 2. Economy cut 3. Token cut Select the correct answer using the code given below Code : (a) 1 and 2 only (b) 2 and 3 only (c) 1, 2and3 (d) I and 3 only 13. Which of the statements given below is/are correct? 1.The Speaker immediately vacates his/her office whenever the State Legislative Assembly is dissolved. 2. No Member of a State Legislative Assembly shall be liable to any proceeding in any court in respect of anything said or any vote given by him/her in the legislature. Select the correct answer using the code given below Code : (a) 1 only (b) 2 only (c) Both 1 and 2 (d) Neither 1 nor 2 14. Consider the following statements about Cripps Proposals of 1942 : 1.Provision was to be made for participation of Indian States in the Constitution-making body. 2. British Government undertook to accept and implement the Constitution. 3. All provinces of British India were to give an undertaking about the acceptance of the Constitution. 4. In the ongoing World War, noresources of British India would be used. Which of the statements given above is/are correct? (a) 1 and 2 only (b) 2 only (c) 1, 2and4 (d) 1, 3 and 4 15. Which one among the following was not one of the causes of the Third Anglo-Maratha War? (a) The Maratha Chiefs,particularly the Peshwa desired to throw off the restrictions imposed on them by the treaties with the English (b) The determination of the English to give a finishing blow to the Maratha power (c) The Pindari designs to seek support from the Marathas to drive the English away from India (d) The attack of Peshwa Baji Rao on the residency at Khirki 16. William Bentinck attempted to reform Hindu society by suppressing/ abolishing social evils. Which one among the following was not included in them? (a) Sati (b) Infanticide (c) Thugi (d) Slavery 17. Karam is a festival celebrated to worship Karam Deuta, the God of power. Which one among the following tribal communities in India traditionally celebrates this festival? (a) Santhals (b) Karbi (c) Meenas (d)Bhils 18. The terra-cotta plough of the Harappan civilization was found at (a) Mohenjo-daro (b) Banawali (c) Kalibangan (d) Lothal 19. Which one among the following works of Mahatma Gandhi provides a critique of modern machine-oriented civilization? (a) The Story of My Experiments with Truth (b) Hind Swaraj (c) Constructive Programme (d) Anasakti Yoga (Commentary on `Gita') 20. The penetrating power of X-rays can be increased by (a) increasing the current in the filament (b) decreasing the potential difference between the cathode and the anode (c) decreasing the current in the filament (d) increasing the potential difference between the cathode and the anode UPSC NDA Mathemtics NDA National Defence Academy Examination General Intelligence Part A April 2011 Original Question Paper with answers and detailed explanations 1. If 9 3 9 = 12 8 2 8 = 12 6 3 6 = ? (1) 8 (2) 9 (3) 5 (4) 12 (5) None of these Ans : (1) Explanation : 9 ÷ 3 + 9 = 12 8 ÷ 2 + 8 = 12 6 ÷ 3 + 6 = 8 2. If 62 + 51 = 16, 91 + 85 = 24, 53 + 82 = 12, 72 + 83 = 25, then 73 + 83 = ? (1) 4 (2) 20 (3) 15 (4) 37 (5) None of these Ans : (2) Explanation : 6 - 2 = 4; 5 - 1 = 4 Now 4 * 4 = 16 Similarly 4 * 5 = 20. 3. Answer in a short-cut method: 709 + 333 + 222 + 111 + 99 + 88 + 77 + 66 + 55 + 44 + 33 + 22 + 11 + 1 = ? (1) 3,001 (2) 1,034 (3) 1,997 (4) 1,871 (5) None of these Ans : (4) Explanation : 709 + 111 + (3 + 2 + 1) + 11 (9 + 8 ... 1) + 1 = 1871 4. Insert the missing number : 2 5 7 4 7 5 3 6 ? (1) 12 (2) 4 (3) 9 (4) 6 (5) None of these Ans : (4) Explanation : (2 + 4) ÷ 2 = 3 (5 + 7) ÷ 2 = 6 ? (7 + 5) ÷ 2 = 6 5. If 6 * 2 = 31, 8 * 4 = 42, 2 * 2 = 11, 6 * 6 = 33, then 8 * 6 = ? (1) 33 (2) 43 (3) 14 (4) 42 (5) None of these UPSC Paper UPSC - IAS Model Preparation Paper 1) In India other than ensuring that public funds are used efficiently and for intended purpose what is the importance of the office of the CAG? 1. CAG exercises exchequer control on behalf of the parliament when the president of India declares national emergency/ financial emergency 2. CAG reports on the execution of projects or programmes by the ministries are discussed by the PAC/ 3. Information form CAG reports can be used by investigating agencies to press charges against those who have violated the law while managing public finances. 4. While dealing with audit and accounting of govt. companies. CAG has certain judicial powers for prosecuting those who violate the law. Which of the above are correct? a.1, 3 and 4 only b.2 only c.2 and 3 only d.1, 2, 3 and 4 Answer: C 2) The endeavour of Janine suraksha yojana programme is 1.to promote institutional deliveries 2.to provide monetary assistance to the mother to meet the cost of delivery 3.to provide for wage loss due to pregnancy and confinements Which of the above are correct? a.1 and 2 only b.2 only c.3 only d.1, 2 and 3 Answer: A 3) The Prime Minister of India, at the time of his/ her appointment a.need not necessarily be a member of one of the Houses of the Parliament but must become a member of one of the Houses within six months b.need not necessarily be a member of one of the Houses of the Parliament but must become a member of the Lok Sabha within six months c.must be a member of one of the Houses of the parliament d.must be a member of the Lok Sabha Answer: A 4. With reference to the Delimitation Commission, consider the following statements: 1. The orders of the Delimitation Commission cannot be challenged in a Court of Law. 2. When the orders of the Delimitation Commission are laid before the Lok Sabha or State Legislative Assembly, they cannot effect any modifications in the orders. Which of the statements given above is/are correct? a. 1 only b. 2 only c. Both 1 and 2 d. Neither 1 nor 2 Answer: C 5. Consider the following: 1.Hotels and restaurants 2.Motor Transport undertakings 3.Newspaper Establishments 4.Private Medical Institutions The Employers of which of the above can have the ? Social Security coverage under Employees State Insurance Scheme? a. 1, 2 and 3 only b. 4 only c. 1, 3 and only 4 d. 1, 2, 3 and 4 Answer: D 6. According to the Constitution of India, it is the duty of the President of India to cause to be laid before the Parliament which of the following? 1.The Recommendations of the Union Finance Commission 2.The Report of the Public Accounts Committee 3.The Report of the Comptroller and Auditor General 4.The Report of the National Commission for the Scheduled Castes Select the correct answer the using the codes given below: a. 1 only b. 2 and 4 only c. 1, 3 and 4 only d. 1, 2, 3 and 4 Answer: C 7. A deadlock between the Lok Sabha and rajyasabha calls for a joint sitting of the parliament during the passage of 1. ordinary legislation 2. money bill 3. constitution amendment bill Select the correct answer using the codes given below. a. 1 only b. 2 and 3 only c. 1 and 3 only d. 1,2 and 3 Answer: A 8. How do District Rural Development Agencies (DRDAs) help in the reduction of rural poverty in India? 1. DRDAs act as Panchayati Raj Institutions in certain specified backward regions of the country. 2. DRDAs undertake area-specific scientific study of the cause of poverty and malnutrition and prepare detailed remedial measures. 3. DRDAs secure inter-sectoral and inter-departmental coordination and cooperation for effective implementation of anti-poverty programmes. 4. DRDAs watch over and ensure effective utilisation of the funds intended for anti-poverty programme. Which of the statements given above is/ are correct? a. 1, 2 and 3 only b. 3 and 4 only c. 4 only d. 1, 2, 3 and 4 Answer: B 9. Which of the following is/ are among the fundamental Duties of citizens laid down in the Indian Constitution? 1. To preserve the rich heritage of our composite culture 2. To protect the weaker sections from social injustice 3. To develop the scientific temper and spirit of inquiry 4. To strive towards excellence in all spheres of individual and collective activity. Select the correct answer using the codes given below: a) 1 and 2 only b) 2 only c) 1, 3 and 4 only d) 1, 2, 3 and 4 Answer: C 10. What is the provision to safeguard the autonomy of the supreme court of India? 1. While appointing the Supreme Court judges, the president of India has to consult the CJI. 2 the SC judges can be removed by the CJI only 3 the salaries of judges are charged on the consolidated fund of India to which the legislature does not have to vote. 4. All appointments of officers and staffs of the SC are made by the govt only after CJI correct a. 1&3 b. 3&4 c. 4 d. 1,2,3,4 Answer: A 11. To meet its rapidly growing energy demand, some opine that India should pursue research and development on thorium as the future fuel of nuclear energy. In this context what advantage does thorium hold over uranium? 1. Thorium is far more abundant in nature than uranium. 2. On the basis per unit mass of mined mineral, thorium can generate more energy compared to natural uranium. 3. Thorium produces less harmful waste compared to uranium. Correct a ) 1 b) 2,3 c) 13 d) 1,2,3 Answer: D 12. The increasing amount of carbon dioxide in the air is slowly raising the temperature of the atmosphere, because it absorbs a) the water vapour of the air and retains its heat b) the UV part of the solar radiation c) all the solar radiations d) the infrared part of the solar radiation Answer: D 13. Which one of the following sets of elements was primarily responsible for the origin of life on the Earth? a) Hydrogen, Oxygen, Sodium b) Carbon, Hydrogen, Nitrogen c) Oxygen, Calcium, Phosphorous d) Carbon, Hydrogen, Potassium Answer: B 14. What are the reasons for the people's resistance to the introduction of BT brinjal in India? 1. BT brinjal has been created by inserting a gene from a soil fungus into its genome. 2. the seeds of BT brinjal are terminator seeds and therefore, the farmers have to buy the seeds before every season from the seed companies. 3. there is an apprehension that the consumption of BT brinjal may have adverse impact on health. 4. there is some concern that the introduction of BT brinjal may have adverse effect on the biodiversity. Correct a 1, 2 and 3 only b 2 and 3 only c 3 and 4 only d 1,2,3 and 4 Answer: D 15. Other than resistance to pests, what are the prospects for which genetically engineered plants have been created? 1. To enable them to withstand drought 2. To increase the nutritive value of the produce 3. To enable them to grow and do photosynthesis in spaceships and space stations 4. To increase their shelf life Answer: c 16. The most effective contribution made by Dadabhai Naoroji to the cause of Indian National Movement was that he 1. Exposed the economic exploitation of India by the British 2. Interpreted the ancient Indian texts stored 3. Stressed the need for eradication of all the social evils before anything else. Which of the statements given above is/ are correct? a) 1 only b) 2 and 3 only c) 1 and 3 only d) 1, 2 and 3 Answer: a 17. With reference to Dhrupad, one of the major traditions of India that has been kept alive for centuries, which of the following statements are correct? 1. Dhrupad originated and developed in the Rajput kingdoms during the Mughal Period. 2. Dhrupad is primarily a devotional and spiritual music. 3. Dhrupad Alap users Sanskrit syllables from Mantras. Select the correct answer using the codes given below. a) 1 and 2 only b) 2 and 3 only c) 1, 2 and 3 only d) 1, 2 and 3 Correct answer: B 18. How do you distinguish between kuchipudi and bharatnatyam dances? 1. Dancers occasionally speaking dialogues i found in kuchipudi dance but not in bharatnatyam. 2 . Dancing on the brass plate by keeping the feet on its edges is a feature of bharatnatyam but have such a form of movements. Correct a) 1 only b) 2 only c) both 1 and 2 d) neither 1 nor 2 Answer: A 20. The Rowlatt act aimed at a) compulsory economic support to war efforts b) imprisonment without trial and summary procedures for trial c) suppression of the khilafat movement d) imposition of restrictions on freedom of the press Answer: B 21. The Lahore session of the Indian nat. congress 1929 is very important in history because 1 the congress passed a resolution demanding complete independence 2 rift between the extremists and moderates was resolved in that session 3 a resolution was passed rejecting two nation theory in that session Which of the statements given above is/ are correct? a) 1 only b) 2 and 3 c) 1 and 3 d) None of the above Answer: A 22. Lord Buddha?s image is sometimes shown with the hand gesture called ?Bhumiparsh Mudra?. it symbolizes a) Buddha?s calling of the earth to watch over Mara and to prevent Mara from disturbing his meditation b) Buddha?s calling of the earth to witness his purity and chastity despite the temptations of Mara c ) Buddha?s reminder to his followers that they all arise from the earth and finally dissolve into the earth and thus this life is transitory d) both a and b are correct. Answer: D 23. The religion of early Vedic Aryans was primarily of a. bhakti b. image worship and yajnas c. worship of nature and yajnas d. worship of nature and bhakti Answer: C 28. Which of the following would include Foreign Direct Investment on India? 1. Subsidiaries of foreign companies in India 2. Majority foreign equity holding in Indian companies 3. Companies exclusively financed by foreign companies 4. Portfolio investment Select the correct answer using the codes given below: a) 1, 2 , 3 and 4 b) 2 and 4 only c) 1 and 3 only d) 1, 2 and 3 only Ans: D 24. Which of the following statements is/ are correct regarding Brahmo Samaj? 1. It opposed idolatry. 2. It denied the need for a priestly class for interpreting the religious texts. 3. It popularized the doctrine that the Vedas are infallible. Select the correct answer using the codes given below: a) 1 only b) 1 and 2 only c) 3 only d) 1, 2 and 3 Answer: B 25. The Reserve Bank of India (RBI) acts as a bankers? bank. This would imply which of the following? 1 Other bank retains their deposits with the RBI. 2 The RBI lends funds to the commercial banks in times of need. 3 The RBI advises the commercial banks on monetary matters. Correct a )2 and 3 only b )1 and 2 only c )1 and 3 only d )1, 2 and 3 Answer: B 26. Under which of the following circumstances may ?capital gains ?arise? 1. when there is an increase in the sales of a product 2 .when there is a natural increase in the value of the property owned. 3. when you purchase a painting and there is a growth in its value due to increase in its popularity. Select the correct answer using the codes given below: a) 1 only b )2 and 3 only c )2 only d )1, 2 and 3 Answer: B 27. Which of the following measures would result in an increase in the money supply in the economy? 1. Purchase of govt securities from the public by the central bank 2. Deposit of currency in commercial banks by the public 3 .borrowing by the govt. from the central bank 4 .Sale of govt. secs. To the public by the central bank Select the correct answer using the codes given below: a )1 b )2 and 4 only c )1 and 3 d )2, 3 and 4 Answer: C 28. Which of the following would include Foreign Direct Investment on India? 1. Subsidiaries of foreign companies in India 2. Majority foreign equity holding in Indian companies 3. Companies exclusively financed by foreign companies 4. Portfolio investment Select the correct answer using the codes given below: a) 1, 2, 3 and 4 b) 2 and 4 only c) 1 and 3 only d) 1, 2 and 3 only Answer: D 29. Consider the following statement: The price of any currency in international market is decided by the 1. World Bank 2. Demand for goods/services provided by the country concerned 3. Stability of the government of the concerned country 4. Economic potential of the country in question. Select the correct answer using the codes given below: a) 1,2 ,3 and 4 b) 2 and 3 only c) 3 and 4 only d) 1 and 4 only Answer: B 30. The basic aim of Lead Bank Scheme is that a) big banks should try to open offices in each district b )there should be stiff competition among the various nationalized banks c )individual banks should adopt particular districts for intensive development d )all the banks should make intensive efforts to mobilize deposits. Answer: C 31. Consider the following: 1. Assessment of the land revenue on the basis of nature of the soil and the quality of crops. 2. Use of mobile cannons in warfare 3. Cultivation of tobacco and red chillies Which of the above was/were introduced into India by the English? a )1 only b )1and 2 c )2and 3 d) None Answer: D 32. With reference to the guilds (Shreni) if ancient India that played a very important role in the country's economy, which of the following statements is/are correct? 1. Every guild was registered with the central authority of the state and the king was the chief admen authority on them 2 .The wages, rules of work, standards and prices were fixed by the guild 3. The guild had judicial powers over its own members. Select the correct answer using the codes given below: a )1 and 2 only b )3 only c )2and 3 only d )1, 2 and 3 Answer: D 33. The distribution of powers between the Centre and the States in the Indian Constitution is based on the scheme provide in the a) Marley-Minto reforms 1909 b)Montagu Chelmsford act 1919 c) Govt. of India act, 1935 d )Indian independence act 1947 Answer: C 34. Despite having large reserves of coal, why does India import millions of tonnes of coal? 1. It is the policy of India to save its own coal reserves for future and import it from other countries for the present use. 2 most of the power plants in India are coal based and they are not able to get sufficient applies of coal from within the country? 3. Steel companies need large quantity of coking coal which has to be imported correct a) 1 only b) 2 and 3 only c) 1 and 3 only d) 1, 2 and 3 Answer: B 35. A person stood alone in a desert on a dark night and wanted to reach his village which was situated 5km east of the point where he was standing .He had no instruments to find the direction but he located the polestar, the most convenient way now to reach his village is to walk in the a) facing the polestar b) opposite to the polestar c) keeping the polestar to his left d) keeping the polestar to his right Answer: C 36. Recently there has been a concern over the short supply of a group of elements called rare earth metals. Why? 1. China, which is the largest producer of these elements, has imposed some restrictions on their export. 2 Other than china, Australia, Canada, Chile, these elements are not found in any country. 3 Rare earth metals are essential for the manufacture of various kinds of electronic items and there is growing demand for these elements. Select the correct answer using the codes given below: a. 1 only b. 2 and 3 only c. 1 and 3 only d. 1, 2 and 3 Answer: C 37) Consider the following protected areas: 1. Bandipur 2. Bhitarkanika 3. Manas 4. Sunderbans Which of the above are declared Tiger Reserves? a) 1 and 2 only b) 1, 3 and 4 only c) 2, 3 and 4 only d) 1, 2, 3 and 4 Answer: B 38. Consider the following statements: 1. The duration of the monsoon decreases from southern India to northern India. 2. The amount of annual rainfall in the northern plains of India decreases from east to west. Which of the statements given above is/are correct? a) 1 only b) 2 only c) Both 1 and 2 d) Neither 1 nor 2 Answer: C 39. Which one of the following is the characteristic climate of the Tropical Savannah Region? a. Rainfall throughout the year b. Rainfall in winter only c. An extremely short dry season d. A definite dry and wet season Answer: D 40. In which one among the following categories of protected areas in India are local people not allowed to collect and use the biomass? a. Biosphere reserves b. National parks c. Wetlands declared under Ramsar convention d. Wildlife sanctuaries Answer: B 41. Consider the following kinds of organisms: 1. Bat 2. Bee 3. Bird which of the above is/are pollinating agent/agents? a) 1 and 2 only b) 2 only c) 1 and 3 only d) 1, 2 and 3 Answer: D 42. Which one of the following groups of animals belongs to the category of endangered species? a. Great Indian bustard, Musk Deer, Red Panda and Asiatic Wild Ass b. Kashmir Stag, Cheetah, Blue Bull, GIB c. Snow Leopard, Swamp Deer, Rhesus Monkey, Saras (Crane) d. Lion Tailed Macaque, Blue Bull, Hanuman Langur, Cheetah Answer: A 43. Consider the following statements: If there were no phenomenon of capillarity 1 it would be difficult to use a kerosene lamp 2 one would not be able to use a straw to consume a soft drink 3 the blotting paper would paper would fail to function 4 the big trees that we see around would not have grown on the earth Which of the following statements given above are correct? a) 1, 2 and 3 only b) 1, 3 and 4 only c) 2 and 4 only d) 1, 2, 3 and 4 Answer: B 44. The Millenium Ecosystem Assessment describes the following major categories of ecosystem services-provisioning, supporting, regulating, preserving and cultural. Which one of the following is supporting service? a) Production of food and water b) Control of climate and disease c) Nutrient Cycling and crop pollinator d) Maintenance of diversity Answer: C 45. What is the difference between the antelopes Oryx and Chiru? a )Oryx is adapted to live in hot and arid areas whereas Chiru is adapted to live in steppes and semi-desert areas of cold high mountains b) Oryx is poached for its antlers whereas Chiru is poached for its musk c )Oryx exists in western India only whereas Chiru exists in north east India only. d )None of the statements (a), (b) and (c) given above is correct. Answer: A 46. Which of the following can be threats to the biodiversity of a geographical area? 1 Global warming 2 Fragmentation of habitat 3 Invasion of alien species 4 Promotion of vegetarianism Select the correct answer using the codes given below: a) 1, 2 and 3 only b) 2 and 3 only c) 1 and 4 only d) 1, 2, 3 and 4 Answer: A 47. Consider the following: 1. Black necked crane 2. Cheetah 3. Flying squirrel 4. Snow leopard Which of the above are naturally found in India? a) 1, 2 and 3 only b) 1, 3 and 4 only c) 2 and 4 only d) 1, 2, 3 and 4 Answer: B 48. Consider the following agricultural practices: 1. Contour bunding 2. Relay cropping 3. Zero tillage In the context of global climate change, which of the above helps/help in carbon sequestration/ storage in the soil? a) 1 and 2 only b) 3 only c) 1, 2 and 3 d) None of them Answer: B 49. What would happen if phytoplankton of an ocean is completely destroyed for some reason? 1. The ocean as a carbon sink would be adversely affected. 2. The food chains in the ocean would be adversely affected. 3. The density of ocean water would drastically decrease. Select the correct answer using the codes given below: a) 1 and 2 only b) 2 only c) 3 only d) 1, 2 and 3 Answer: A 50. Vultures used to be very common in Indian countryside some years ago are rarely seen nowadays. This is attributed to a) the destruction of their nesting sites by new invasive species b) a drug used by cattle owners for treating their diseased cattle c) Scarcity of food available to them d) a widespread, persistant and fatal disease among them Answer: B UPSC Aptitude-General CAPF Model Question Paper 1. The sum of the base and altitude of a triangle is 30 cm. What is the maximum possible area of such a triangle? (a) 100cm2 (b) 110cm2 (c) 112.5 cm2 (d) 120 cm2 2. Through how many degrees does the hour hand in a clock move as the time changes from 3 hours and 12 minutes to 6 hours? (a) 105 (b) 99 (C) 90 (d) 84 3. If 5 persons can weave 160 mats in 8 days, how many mats will 8 persons weave in 6 days? (a) 200 (b) 192 (c) 190 (d) 180 4. Two cars are moving in the same direction with a speed of 45 km/hr and a distance of 10 km separates them. If a car coming from the opposite direction meets these two cars at an interval of 6 minutes, its speed would be (a) 45 km/hr (b) 55 km/hr (c) 65 km/hr (d) 75 km/hr 5. The least integer whose multiplication with 588 leads to a perfect square is (a) 2 (b) 3 (c) 4 (d) 7 6. The fifth term of the alphabet series BCYX, EFVU, HISR, KLPO,..is (a) NOML (b) NOLM (c) ONML (d) ONLM 7. By selling an article at f 270, a man loses 10%. If he would sell it at 360, his gain percent is (a) 10 (b) 15 (c) 20 (d) 25 8. In a certain code, 'PLANT' is written as '$@2*©' and 'YIELD' is written as 1364@%'. How is 'DELAY' written in that code? (a) f34*2% (b) 134@2% (c) %42@3 (d) %4@211 9. If A is coded as 1, B as 3, C as 5 and so on, which of the following is the numerical value of the word `FAZED' if the numerical value of 'CABLE' is 41? (a) 81 (b) 80 (c) 79 (d) 77 10. Natural gas is a mixture of gases and contains mainly (a) methane and higher hydro-carbons (b) butane and isobutene (c) methane only (d) methane, hydrogen and carbon monoxide 11. Diamond is a polymorph of graphite.Both contain carbon atoms, but they have extremely different properties because of the condition in which they are formed. Diamond is obtained after applying (a) very high pressure and low temperature (b) very low pressure and high temperature (c) very low pressure and low temperature (d) very high pressure and high temperature 12. The three primary soil micro-nutrients are (a) carbon, oxygen and water (b) copper, cadmium and carbon (c) nitrogen, phosphorus and potassium. (d) boron, zinc and magnesium 13. Diamond Ring, God's Eye and Baily's Beads are the parts of which one among the following natural phenomena? (a) Solar eclipse (b) Aurora (c) Lightning (d) Solar storm 14. 'Turing machine' serves as (a) a machine to identify explosives (b) an indicator to inform about forest fire (c) an instrument for finding out gravitational constant (d) a theoretical computing machine to act as an ideal model for mathematical calculation 15. Leaves of lotus and water lily are not easily wet because the leaves (a) have surface uneven in micro-scale and water cannot come into contact with the depressed areas due to high surface tension (b)contain an oily substance (c) contain a greasy substance (d) have surface too smooth to attract water 16. Leaves of lotus and water lily are not easily wet because the leaves (a) have surface uneven in micro-scale and water cannot come into contact with the depressed areas due to high surface tension (b)contain an oily substance (c) contain a greasy substance (d) have surface too smooth to attract water 17. Arsenic present in groundwater can be partially removed (a) along with removal of iron by precipitation/ coagulation method if iron is also present in the water (b) on boiling even if iron is absent in the water (c) on UV-treatment (d) on filtration even if iron is absent in the water 18. Transformer is used to 1. convert low a.c. voltage to high voltage 2. convert high a.c. voltage to low voltage 3. convert direct current to alternating current 4. regulate the fluctuation of voltage Select the correct answer using the code given below. (a) 1 and 3 only (b) 3 and 4 (c) 1 and 2 only (d) 1, 2 and 3 19. The filament of electric bulb is generally made of tungsten because (a) tungsten is cheap (b) durability of tungsten is high (c) light-emitting power of tungsten is high (d) melting point of tungsten is high 20. Electricity meter may be a/an (a) mainframe. computer (b) microcomputer (c) hybrid computer (d) analog computer 21. Advanced sunrise and delayed sun-set found in the sky are due to the phenomenon of (a) diffraction of sunlight (b) refraction of sunlight (c) scattering of sunlight (d) total internal reflection of sun-light 22. The formation of colours in soap bubbles is due to the phenomenon of (a) dispersion of light (b) interference of light (c) diffraction of light (d) polarization of light 23. Geostationary satellite completes its one revolution around the earth in (a) 90 minutes (b) 24 hours (c) 30 days (d) 365 days 24. What is the correct sequence of conductivity when arranged in ascending order? (a) Metals, semiconductors, insulators (b) Insulators, metals, semi-conductors (c) Semiconductors, metals,insulators (d) Insulators, semiconductors,metals 25. Which one among the following statements about `modem' is correct? (a) Modem is a software (b) Modem helps in stabilizing the voltage (c) Modem is an operating system (d) Modem converts the analog signal into digital signal and vice versa 26. Which one among the following is not a form of carbon? (a) Graphene (b) Graphite (c) Fullerene (d) Quartz 27. Plants capable of performing photo-synthesis belong to which one among the following types of organisms? (a) Heterotrophs (b) Saprotrophs (c) Autotrophs (d) Chemoheterotrophs 28. With reference to India, which of the following statements relating to urbanization is/are correct? 1., It is a manifestation of economic, social and political progress. 2. Rapid growth of urban population puts pressure on public utilities. 3. Urban sprout is encroaching the precious agricultural land. Select the correct answer using the code given below. (a) 1 and 2 only (b) 2 only (c) 1 and 3 only (d) 1, 2 and 3 29. Which of the following statements regarding landslides is/are correct? 1. They are sudden and sporadic. 2. They result when the slope is gentle. 3. Mining activity is also responsible for landslides. Select the correct answer using the code given below. (a) 1 and 2 only (b) 1 and 3 only (c) 3 only (d) 1, 2 and 3 30. Which of the following statements related to globalization is/are correct? 1. It has resulted in the change of scale in economic activities. 2. End of Cold War facilitated globalization. 3. Flow of money in transnational corporations is discouraged. Select the correct answer using the code given below. (a) 1 and 2 only (b) 1 only (c) 2 and 3 only (d) 1, 2and 3 31. The continent having the largest area in the equatorial belt is (a)Asia (b) South America (c) Africa (d) North America 32. Which of the following statements are correct? 1. Sediments of Gondwana System were deposited under marine condition. 2. Rocks of Gondwana System are fossiliferous. 3. Rocks of Gondwana System contain metallic mineral deposits. 4. Gondwana deposition took place in downfaulted trough. Select the correct answer using the code given below. (a) 1, 2and 3 (b) 1, 2and 4 (c) 2 and 4 only (d) 1 and 3 only 33. Which one amongthefollowing statements is not correct? (a)Shillong Plateau is an outlier of Peninsular Plateau of India (b) Aravalli Mountain is the oldest mountain chain of India (c) Vindhyans are examples of fold mountain (d) Rajmahal Highlands are composed of lava deposits 34. Which one among the following systems contains the oldest rock formation of India? (a) Aravalli System (b) Archean System (c) Cuddapah System (d) Vindhyan System 35. Which one among the following is not correct regarding the black soils of Peninsular India? (a) Rich in iron, lime and potash (b) Retentive in moisture (c) Fertile and well-suited for growing sugarcane (d) Rich in phosphorus and organic matter 36. Which one among the following places in India is . nearest to the Tropic of Cancer? (a) Aizawl (b) Rajkot (c) Kolkata (d) Imphal 37. Which one among the following States shares common boundaries with maximum number of other States? (a) Madhya Pradesh (b) West Bengal (c) Chhattisgarh (d) Andhra Pradesh 38. The State of Sikkim is surrounded by (a) China, Nepal, Bhutan and West Bengal (b) Bhutan, Nepal, West Bengal and .Assam (c) China, Nepal, West Bengal and Assam (d) China, Bhutan, West Bengal and Assam 39. Khumbum monastery is situated in (a)India (b) Thailand (c)China (d) Japan 40. The place located at the confluence of Alakananda and Pindar rivers is (a) Rudraprayag (b) Karnaprayag (c) Devaprayag (d) Vishnuprayag 41. Which one among the following ocean currents mixes with Labrador current? (a)Canaries current (b) Benguela current (c) Florida current (d) California current 42. The Wallace's Line distinguishes or separates the flora and fauna between (a) Canada and USA (b) Taiga region and Tundra region (c) Mexico and Central America (d) South-East Asia and Australasia 43. Which one among the following is the major occupation in equatorial region? (a)Growing of tropical crops (b)Growing of cereals (c) Growing of citrus fruits (d) Animal rearing 44. Mahatma Gandhi's opposition to separate electorates for untouchables was because (a) separate electorates would ensure them bondage in perpetuity (b) of the apprehension of offending the conservative high caste Hindus (c) it would. weaken Mahatma Gandhi's position as a leader (d) None of the above 45. Various causes have been suggested for the decline of Harappan culture.Consider the causes mentioned below and identify the weakest of them. (a) Due to constant incidents of naturally occurring floods (b) Due to increase in the salinity of the soil, fertility decreased there (c) Earthquakes caused changes in the course of the Indus,which led to inundation (d) The Aryans invaded and destroyed the Harappan culture 46. Which one among the following inscriptions was installed in the easternmost part of Ashoka's territory? (a) Barabar hill cave inscription (b) Dhauli major rock edicts (c) Sahasram minor rock edicts (d) Lauriya Araraj pillar edicts 47. The prevalence of grain in Harappan sites indicated that (a) Harappans ploughed their fields (b) Harappans broadcast seed on their fields (c) agriculture was prevalent in Harappan sites (d) trade in bulk items like food grains was well-established in Harappan times 48. Which one among the following statements about industries in colonial India is not correct? (a) The jute industry was almost entirely owned by Europeans (b) The cotton textile industry was almost wholly owned by Indians (c) The cotton textile industry was small in size (d) The growth of. cotton industry was the result of government support 49. The establishment of Mughal rule in India 1. strengthened urbanization 2. strengthened inter-urban contact in riparian North India 3. increased India's sea trade 4. decreased the risk of carrying long-distance trade Select the correct answer using the code given below. (a) 1, 2and3 (b) 1 and 2 only (c) 3 and 4 (d) 1 only 50. The first major boost to modern industry in India happened in the 1860s. It was a direct result of (a) the demand created in America because of the Civil War (b) the demand created in Europe because of the Civil War in America (c) the decline in competition from the cotton mills of Lancashire and Manchester (d) the support of government to industries in India UPSC Aptitude questions and answers UPSC CDS elementary mathematics,algebra stastitics solved question papers of previous years,UPSC CDS General awareness questions with answers and detailed explanations 1. Find the H.C.F and L. C. M of x4y6z7 and x3y8z5 ----- (a) x3y6z5, x4y8z7 - Ans (b) x4y6z7, x3y8z5 (c) x3y6z7, x4y8z5 (d) xy2z2, x4y6z5 2. If a Superfast train running at the rate of 120km per hour passes a bridge of 100metres long in 12 seconds, what is the length of the train ? (a) 260 m (b) 300 m - Ans (c) 360 m (d) 400 m Expl : Distance covered by train in 12 seconds = 120 * 5/18 * 12 = 400 m :. Length of the train = 400-100 = 300 m 3. A tradesman marks his goods 30% above the C.P. If he allows discount of 6 and 1/4 %, then his gain percent is : (a) 23 and 3/4 % (b) 22 % (c) 21 and 7/8% - Ans (d) None Expl : Let CP. be Rs.100. Then, marked price = Rs.130 S.P. = (100 - 25)% of Rs.130=(375 x130) = 121.875 4 400 :. Profit % = 21.875 = 21875 = 21 and 7/8 % 1000 4. A sum of money becomes 8 times of itself in 3 years at compound interest. The rate of interest is : (a) 100 % - Ans (b) 8 % (c) 1 % (d) Data inadequate Expl : P ( 1+ R )3 = 8P or (1+ R )3 = 8 = 23 100 100 :. 1+ R = 2 or R = 1 or R = 100 100 100 5. A and B are two alloys of gold and copper prepared by mixing metals in the ratio 7:2 and 7:11 respectively. If equal quantities of the alloys are melted to form a third alloy C, the ratio of gold and copper in C will be : (a) 5:9 (b) 5:7 (c) 7:5 - Ans (d) 9:5 Expl : Gold in C (7/9 + 7/18) = 7/6 Copper in C = (2/9 + 11/18) = 5/6 Gold : Copper = 7/6 : 5/6 : = 7 : 5 6. Three years ago the average age of A and B was 18 years. With C joining them now, the average becomes 22 years. How old is C now ? (a) 24 years - Ans (b) 27 Years (c) 28 years (d) 30 years Expl : (A+B)'s age 3 years ago = (18 x2) years = 36 years. :. (A+B) now = (36+3+3+) years = 42 years (A+B+C) now = (22x3) years = 66 years :. C now = (66-42) years = 24 years 7. The value of (2n + 2n-1) is : ________ 2n+1 2n (a) 1/2 (b) 3/2 - Ans n-1 (c) 2 n+1 (d) None of these Expl: Given exp. = 2 n-1 (2+1) 3 _________ = 2 n(2-1) 2 8. On dividing 4150 by a certain number, the quotient is 55 and the remainder is 25. The divisor is : (a) 65 (b) 70 (c) 75 - Ans (d) 80 Expl : Divisor = (Dividend - Remainder) = (4150 - 25) = 75 Quotient 55 9. 0.000006723 when expressed in scientific notation, is : (a) 6723 x 10 -5 (b) 67.23 x 10 -7 (c) 6.723 x 10 -6 - Ans (d) None Expl : 0.000006723 = 0.000006723 x 106 = 6.723 = 6.723x 10-6 106 106 10. A man distributed Rs.100 equally among his friends. If there had been 5 more friends, each would have received one rupee less. How many friends had he ? (a) 20 - Ans (b) 25 (c) 30 (d) 35 Expl : Suppose he had x friends. Then, 100 - 100 = 1 or 1 - 1 = 1 x x+5 x x+5 100 x+5-x = 1 or x2+5x-500 = 0 or (x+25)(x-20) = 0 x(x+5) 100 :. x = 20 11. ? 9.5 x .0085 x18.9 is equal to : .0017 x 1.9 x 2.1 (a) 0.15 (b) 0.5 (c) 15 - Ans (d) 250 Expl : Given Exp = ? 95 x 85 x 189 = ? 5x5x9 = 15 17 x 19 x 21 12. The sum of two numbers is twice their difference. If one of the numbers is 10, the other one is : (a) 3 and 1/3 (b) -3 and 1/3 (c) 4 and 1/4 (d) 30 - Ans Expl : Let the other number be x. Then, 10 + x = 2 (x-10) => x = 30 13. A cricket team won 40% of the total number of matches it played during a year. If it lost 50% if the matches played and 20 matches were drawn, the total number of matches played by the team during the year was : (a) 200 - Ans (b) 100 (c) 50 (d) 40 Expl: 40 % of x + 50% of x + 20 = x => 40 x + 50x + 20 = x or x =200 100 100 14. A workers earn Rs.280 in 5 days working 8 hours per day. How much will 12 workers earn in 2 days working 4 hours per day ? (a) Rs.168 - Ans (b) Rs.172 (c) Rs.224 (d) Rs.196 Expl : If a workers working 8 hours per day earn in 5 days Rs.280 :. I worker working 1 hour per day earn in 1 day = 280 4x8x5 :. 12 workers working 4 hours per day earn in 2 days = 280 x 12 x 4 x 2 = Rs.168 4 x 8 x 5 15. A thief steals a car at 2.30 pm and drives it at 60 kmph. The theft is discovered at 3 pm, and the owner sets off in another car at 75 kmph. When will he overtake the thief? (a) 4.30 p.m. (b) 4.45 p.m. (c) 5.00 p.m. - Ans (d) 5.15 p.m. Expl : Suppose the thief is overtaken x hrs after 2.30pm. Then, distance covered by the thief in x hrs. =distance covered by the owner in (x-1/2)hrs :. 60 x = 75(x-1/2) or 15x = 75/2 or x=5/2 hrs. ie., He is overtaken at 5 p.m. 16. A toothed wheel of diameter 50 cm is attached to a smaller wheel of diameter 30cm. How many revolutions will the smaller wheel make when the larger one makes 15 revolutions? (a) 18 (b) 20 (c) 25 - Ans (d) 30 Expl : Distance moved by toothed wheel in 15 revolutions = (15x2x22/7x25)cm Distance moved by smaller wheel in 1 revolution = (2x22/7x15)cm :. Required number of revolutions = (15 x 44/7 x 25x 7 ) = 25 44 x 15 17. The number of rounds that a wheel of diameter 7/11m will make in going 4km, is : (a) 1000 (b) 1500 (c) 1700 (d) 2000 - Ans Expl : Distance moved in 1 revolution = (2 x 22/7 x 7/22)m = 2m Number of revolutions in going 4 km = (4 x 1000) = 2000 2 18. A tap can fill a tank in 16 minutes and another can empty it in 8 minutes. If the tank is already half full and both tanks are opened together, the tank will be : (a) filled in 12 min. (b) emptied in 12 min. (c) filled in 8 min (d) emptied in 8 min - Ans Expl : Rate of waste pipe being more, the tank will be emptied when both the pipes are opened Net emptying work done by both in 1 min. = (1/8 - 1/16) = 1/16 Now, full tank will be emptied by them in 16 min. :. Half full tank will be emptied in 8 min. 19. The length of the longest rod that can be placed in a room 30m long, 24 m broad and 18m high, is : (a) 30m (b) 15?2m (c) 30?2m - Ans (d) 60 m Expl : length of the longest rod = length of diagonal = ?l2 + b2+h2 = ?302 + 242 + 182 = ?1800 = 30?2 m 20. A cube of side 6cm is cut into a number of cubes, each of side 2cm. The number of cubes will be : (a) 6 (b) 9 (c) 12 (d) 27 - Ans Expl : Number of cubes = Volume of bigger cube = (6 x6x6) = 27 Volume of smaller cube 2x2x2 21. ABC is a triangle right-angled at C. If p is the length of the perpendicular from C to AB and a,b,c, are the sides, then which one of the following is correct ? (a) pa = bc (b) pb =ca (c) pc=ab - Ans (d) p2 =ab Expl : Area of triangle ABC = 1/2 x b x a = 1/2 x p x c :. pc = ab 22. What is the value of ------- (a) 2 (b) >2 (c) >2 - Ans (d) <2 Directions (q.23.25) : The students of various classes of a school, are represented by the following Pie-chart. Total number of students of the school is 1000. Answer each of the following questions on the basis of the pie-chart. 23. How many number of students of class VI is less than number of students of class IX ? (a) 133 (b) 57 (c) 166 (d) 67 - Ans (e) None of these Expl : No of students in a class VI = 72 x 1000 = 200 360 and no of students of class IX = 96 x 1000 = 267 360 :. Reqd difference = 267-200 = 67 24. What is the ratio between the number of students of class X and the number of students of class VIII ? (a) 2 :3 (b) 4:3 (c) 1:2 (d) 4:5 (e) 9:4 - Ans Expl : No of students in a class X = 108 x 1000 = 300 360 and no of students of class VIII = 48 x 1000 = 400 360 3 :. Reqd ratio = 300 : 400/3 = 9:4 25. How many per cent is the number of students of class VII to the total number of student of the school ? (a) 36 (b) 71 (c) 25 (d) 10 - Ans (e) None of these Expl : No of students in a class VII = 36 x 1000 = 100 360 :. Reqd percentage = 100x100 = 10 1000 UPSC CDS English Questions UPSC CDS Practice English Questions and answers One word Substitution Directions - Below are given some statements each of which is capable of being expressed by one word only. Each statement is followed by four such words of which only one is correct. Your task is to pick out the correct choice. 1. A person who renounces the world and practices self-discipline in order to attain salvation - (A) Sceptic (B) Ascetic (Ans) (C) Devotee (D) Antiquarian 2. One who abandons his religious faith- (A) Apostate (Ans) (B) Postate (C) Profane (D) Agnostic 3. A hater of learning and knowledge- (A) Bibliophile (B) Philologist (C) Misogynist (D) Misologist (Ans) 4. Commencement of words with the same letter - (A) Pun (B) Transfered epithet (C) Oxymoron (D) Alliteration (Ans) 5. A person who does not believe in the existence of God- (A) Theist (B) Heretic (C) Atheist (Ans) (D) Fanatic 6. A lady's umbrella is - (A) Parasol (Ans) (B) Granary (C) Epitaph (D) Aviary (E) None 7. Story of old time gods or heroes is- (A) Lyric (B) Epic (C) Legend (Ans) (D) Romance (E) None 8. A sad song is a- (A) Ditty (B) Knell (C) Dirge (Ans) (D) Lay (E) None 9. One who believes in the power of fate - (A) Fatalist (Ans) (B) Optimist (C) Pessimist (D) Parsimonious (E) Pusillanimous 10. A person who loves everybody- (A) Cosmopolitan (Ans) (B) Fratricide (C) Atheist (D) Aristocrat 11. One who cannot be easily pleased- (A) Cosmopolitan (B) Fratricide (C) Fastidious (Ans) (D) Feminist 12. The murder of brother- (A) Homicide (B) Fratricide (Ans) (C) Dermatologist (D) Regicide 13. A person who is indifferent to pains and pleasure of life- (A) Stoic (Ans) (B) Sadist (C) Psychiatrist (D) Patricide 14. Talking to one's own self- (A) Sinecure (B) Soliloquy (Ans) (C) Pessimist (D) Philanthropist 15. A disease which spreads by contact- (A) Incurable (B) Infectious disease (C) Contagious disease (Ans) (D) Fatal disease 16. A man who rarely speaks the truth- (A) Crook (B) Liar (C) Scoundrel (D) Hypocrite (Ans) 17. A remedy for all diseases- (A) Narcotics (B) Antiseptic (C) Panacea (Ans) (D) Tonic 18. International destruction of racial groups- (A) Matricide (B) Regicide (C) Genocide (Ans) (D) Homicide 19. Custom of having many wives- (A) Matrimony (B) Bigamy (C) Polygamy (Ans) (D) Monogamy 20. A pioneer of a reform movement- (A) Apostle (Ans) (B) Apothecary (C) Apotheosis (D) Renegade 21. A person who pretends to have more knowledge or skill than he really has- (A) Apostle (B) Apotheosis (C) Renegade (D) Charlatan (Ans) 22. A person who forsakes religion- (A) Apostle (B) Apotheosis (C) Renegade (Ans) (D) Charlatan 23. One who studies insect life- (A) Geologist (B) Zoologist (C) Entomologist (Ans) (D) Botanist 24. A person who does not believe in the existence of God- (A) Atheist (Ans) (B) Chaperon (C) Sycophant (D) Parasite 25. A doctor who treats skin diseases- (A) Opthalmologist (B) Dermatologist (Ans) (C) Paediatrician (D) Cardiologist UPSC CDS English Questions UPSC CDS Practice English Questions and answers One word Substitution Directions - Below are given some statements each of which is capable of being expressed by one word only. Each statement is followed by four such words of which only one is correct. Your task is to pick out the correct choice. 1. A person who renounces the world and practices self-discipline in order to attain salvation - (A) Sceptic (B) Ascetic (Ans) (C) Devotee (D) Antiquarian 2. One who abandons his religious faith- (A) Apostate (Ans) (B) Postate (C) Profane (D) Agnostic 3. A hater of learning and knowledge- (A) Bibliophile (B) Philologist (C) Misogynist (D) Misologist (Ans) 4. Commencement of words with the same letter - (A) Pun (B) Transfered epithet (C) Oxymoron (D) Alliteration (Ans) 5. A person who does not believe in the existence of God- (A) Theist (B) Heretic (C) Atheist (Ans) (D) Fanatic 6. A lady's umbrella is - (A) Parasol (Ans) (B) Granary (C) Epitaph (D) Aviary (E) None 7. Story of old time gods or heroes is- (A) Lyric (B) Epic (C) Legend (Ans) (D) Romance (E) None 8. A sad song is a- (A) Ditty (B) Knell (C) Dirge (Ans) (D) Lay (E) None 9. One who believes in the power of fate - (A) Fatalist (Ans) (B) Optimist (C) Pessimist (D) Parsimonious (E) Pusillanimous 10. A person who loves everybody- (A) Cosmopolitan (Ans) (B) Fratricide (C) Atheist (D) Aristocrat 11. One who cannot be easily pleased- (A) Cosmopolitan (B) Fratricide (C) Fastidious (Ans) (D) Feminist 12. The murder of brother- (A) Homicide (B) Fratricide (Ans) (C) Dermatologist (D) Regicide 13. A person who is indifferent to pains and pleasure of life- (A) Stoic (Ans) (B) Sadist (C) Psychiatrist (D) Patricide 14. Talking to one's own self- (A) Sinecure (B) Soliloquy (Ans) (C) Pessimist (D) Philanthropist 15. A disease which spreads by contact- (A) Incurable (B) Infectious disease (C) Contagious disease (Ans) (D) Fatal disease 16. A man who rarely speaks the truth- (A) Crook (B) Liar (C) Scoundrel (D) Hypocrite (Ans) 17. A remedy for all diseases- (A) Narcotics (B) Antiseptic (C) Panacea (Ans) (D) Tonic 18. International destruction of racial groups- (A) Matricide (B) Regicide (C) Genocide (Ans) (D) Homicide 19. Custom of having many wives- (A) Matrimony (B) Bigamy (C) Polygamy (Ans) (D) Monogamy 20. A pioneer of a reform movement- (A) Apostle (Ans) (B) Apothecary (C) Apotheosis (D) Renegade 21. A person who pretends to have more knowledge or skill than he really has- (A) Apostle (B) Apotheosis (C) Renegade (D) Charlatan (Ans) 22. A person who forsakes religion- (A) Apostle (B) Apotheosis (C) Renegade (Ans) (D) Charlatan 23. One who studies insect life- (A) Geologist (B) Zoologist (C) Entomologist (Ans) (D) Botanist 24. A person who does not believe in the existence of God- (A) Atheist (Ans) (B) Chaperon (C) Sycophant (D) Parasite 25. A doctor who treats skin diseases- (A) Opthalmologist (B) Dermatologist (Ans) (C) Paediatrician (D) Cardiologist UPSC CDS General Knowledge Question for Practice UPSC GENERAL-AWARENESS Practice Questions with answers of Current Events 2013,General Knowledge Questions with answers Objective Current Affairs Q. China recently became ________ arms exporter. 1 The world's second largest 2 The world's fourth largest 3 The world's fifth largest 4 The world's sixth largest Ans: 3 Q. Which research centre announced that it was developing the largest magnet of the world (weight would be 50000 tons)? 1 NASA, USA 2 Bhabha Atomic Research Centre, India 3 CERN, Geneva 4 ISRO, India Ans: 2 Q. Which compost plant become the first in India to receive carbon credit worth Rs.25 lakh from the United Nations Framework Convention on Climate Change? 1 Mumbai Compost Plant 2 Kanpur Composting PLant 3 Rotopala Composting Plant 4 Okhla Compost PLant Ans: 4 Q. Which Malaysian budget carrier to launch the new airline along with Tata Group and Telestra Tradeplace? 1 Jet Star 2 Tiger Airways 3 Air Asia 4 Cebu Pacific Ans: 3 Q. Name the two PSUs, which were granted the Maharatna status by the Union Government of India in February 2013? 1 ONGC and NTPC 2 BHEL and GAIL 3 ONGC and GAIL 4 Coal India and SAIL Ans: 2 Q. Who has won the "Entrepreneur of the Decade" award conferred by All India Management Association in New Delhi? 1 Mr. Ratan Tata 2 Mr. Narayan Murti 3 Mr. Mukesh Ambani 4 Mr. Sunil Mittal Ans: 3 Q. Who is the winner of prestigious Jnanpith award for the 2012? 1 Amar Kant (Hindi) 2 Dr. Ravuri Bharadwaja (Telgu) 3 Pratibha Ray (Odia) 4 Shrilal Shukla (Hindi) Ans: 2 Q. The proposed allocation for the newly launched "National Livestock Mission" in the Union Budget 2013-14 is 1 Rs.304 crore 2 Rs.300 crore 3 Rs.310 crore 4 Rs.307 crore Ans: 4 Q. To encourage farmers to switch to alternative crops from rice and wheat, the Budget 2013-14 has pledged 1 Rs.700 crore 2 Rs.600 crore 3 Rs.500 crore 4 Rs.400 crore Ans: 3 Q. The Budget 2013-14 has proposed to allocate for urban transportation under Jawaharlal Nehru National Urban Renewal Mission (JNNURM) a sum of 1 Rs.14,873 crore 2 Rs.13,873 crore 3 Rs.12,873 crore 4 Rs.10,873 crore Ans: 1 Q. In the Budget 2013-14, how much money has been allocated to the Ministry of Science and Technology? 1 Rs.5,275 crore 2 Rs.7,275 crore 3 Rs.6,275 crore 4 Rs.8,275 crore Ans: 3 Q. How much money has been allocated to the Ministry of Minorities Affairs in the Budget 2013-14? 1 Rs.3,912 crore 2 Rs.3,511 crore 3 Rs.5,511 crore 4 Rs.4,511 crore Ans: 2 Q. Agriculture and allied activities, according to the Economic Survey 2012-13, accounted for Gross Domestic Products 1 14.1 % 2 13 % 3 14 % 4 14.2 % Ans: 1 Q. The Railway Budget 2013-14 has proposed to introduce a new hyper luxury class to be called 1 Udaan 2 Anand 3 Apoorva 4 Anubhuti Ans: 4 Q. According to the 12th Five-Year Plan, India should bring Infant Mortality Rate by the end of the Plan down to 1 27 2 28 3 26 4 25 Ans: 2 Q. By the end of the Plan, the 12th Five-Year Plan aims to train under the Skill Development Mission 1 40 million individuals 2 30 million individuals 3 50 million individuals 4 45 million individuals Ans: 3 Q. The projected average level of the domestic savings rate for the 12th Plan is 1 35.2 % 2 34.6% 3 34.2 % 4 30.1 % Ans: 3 Q. According to the Census 2011, the percentage of Urban Indians connected to a piped sewer system is 1 32.7 % 2 34.6 % 3 46.4 % 4 50.3 % Ans: 1 Q. Which film has bagged the National Film Award for best feature film? 1 Anumati 2 Paan Singh Tomar 3 Dhag 4 Spirit Ans: 2 Q. Who has won the National Film Award as best actor? 1 Irrfan Khan & Vikram Gokhale 2 Shivaji Lotan 3 Kamal Hasan 4 Ranbir Kapoor Ans: 1 UPSC Solved Question Papers Previous years UPSC Question Papers 1. The Headquarters of International Atomic Energy Agency is in ------- (a) Geneva (b) Paris (c) Vienna - Answer (d) Washington 2. In the Budget Estimates for 2011-12, an allocation of Rs.400 crore has been made to bring in a green revolution in the East in the rice-based cropping system of (a) Assam & West Bengal only (b) Assam, West Bengal, Odisha & Bihar only (c) Assam, West Bengal, Odisha, Bihar & Jharkhand only (d) Assam, West Bengal, Odisha, Bihar , Jharkhand, Eastern Uttar Pradesh & Chattisgarh - Answer 3. If the Budget for 2011-12 presented by the Finance Minister on 28.02.2011, the income-tax exemption limit for senior citizens (60 years to less than 80 years) has been increased to ------ (a) Rs. 2.50 lakh - Answer (b) Rs.2.60 lakh (c) Rs.2.80 lakh (d) Rs.3.00 lakh 4. If the Anglo-Indian community does not get adequate representation in the Lok Sabha, two members of the community can be nominated by the (a) Prime Minister (b) President - Answer (c) Speaker (d) President in consultation with the Parliament 5. For the election of President of India, a citizen should have completed the age of ------ (a) 25 Years (b) 30 Years (c) 35 Years - Answer (d) 18 Years 6. Who said, "A good citizen makes a good State and a bad citizen makes a bad State ". (a) Plato (b) Rousseau (c) Aristotle - Answer (d) Laski 7. A member of Parliament will lose his membership of Parliament if he is continuously absent from sessions for (a) 45 days (b) 60 days - Answer (c) 90 days (d) 365 days 8. In India, the Residuary Powers are vested with ------ (a) Union Government - Answer (b) State Government (c) Both the Union Government and the State Government (d) Local Government 9. Mention the place where Buddha attained enlightenment ----- (a) Sarnath (b) Bodh Gaya - Answer (c) Kapilavastu (d) Rajgriha 10. 'World Environment Day' is celebrated every year on ---- (a) 15 January (b) 26 August (c) 5 June - Answer (d) 10 July (e) 20 July 11. If x + 2 =10, then the value of x is ---- 3+ 4 5+ 7 6 (a) 1276/135 - Answer (b) 53/6 (c) 4.35 (d) 9 Expl: x + 2 = 10 3+ 4 5+ 7 6 => x + 2 =10 3+ 4 37 6 => x + 2 =10 3+ 2 4 37 =>x + 2 =10 135 37 =>x + 74 =10 135 :. x = 10-74/135 = 1350-74 = 1276 135 135 12. The unit digit in the sum of (124)372 + (124)373 is ----- (a) 5 - Answer (b) 4 (c) 2 (d) 0 Expl : = (124)372 (1+ 124) = (124)372 x 125 = (4)o x 125 :. Required unit digit = 5 13. The value of 3 + 1 + 1 + 1 is ------ ? 3 3+? 3 ? 3 - 3 (a) 3+? 3 (b) 3 - Answer (c) 1 (d) 0 Expl : 3 + 1 + ? 3-3+3+? 3 ? 3 (3+? 3 )(? 3 -3) = 3 + 1 + 2? 3 ? 3 (-6) = 3 + 1 - 1 ? 3 ? 3 = 3 14. If a * b = a, then the value of 5*3 is ---- (a) 125 - Answer (b) 243 (c) 53 (d) 15 Expl: a * b = a 5 * 3 = 53 = 125 15. If 1.5x = 0.04y, then the value of y2 - x2 is y2+2xy+x2 (a) 730/77 (b) 73/77 - Answer (c) 73/770 (d) 74/77 Expl : 1.5x = 0.04y => y = 1.5 =150 = 75 ----(1) x 0.04 4 2 Given exp. = y2 - x2 is ( y2+2xy+x2 ) = (y-x) (y+ x) = (y - x) (y+x)2 (y + x) = (y/x-1) y/x+1) (Put the value of x =75/2) = 75/2-1 = 73 75/2+1 77 16. If ? 0.03 x0.3xa = 0.3 x 0.3 x ? b value of a/b is ------ (a) 0.009 (b) 0.03 (c) 0.9 - Answer (d) 0.08 Expl : => 0.009 a = (0.09)2 b => 0.009 a = 0.0081 b :. a/b = 0.0081/0.009 = 81/90 = 0.9 17. The price of a commodity rises from Rs.6 perkg to Rs.7.50 per kg. If the expenditure cannot increase, the percentage of reduction in consumption is ----- (a) 15 (b) 20 - Answer (c) 25 (d) 30 Expl : 7.50 - 6.00 = Rs.1.50 :. Required % of reduction = 1.50 x 100 % 7.50 = 20% 18. First and second numbers are less than a third number by 30% and 37% respectively. The second number is less than the first by ---- (a) 7% (b) 4% (c) 3% (d) 10% - Answer Expl : Let the third number be 100 First number = 70 and the second number = 63 :. Reqd. % = 70-63 x 100% = 10% 70 19. Walking at 6/7th of his usual speed a man is 25 minutes too late. His usual time to cover this distance is ----- (a) 2 hours 30 minutes - Answer (b) 2 hours 15 minutes (c) 2 hours 25 minutes (d) 2 hours 10 minutes Expl : Let the usual time be x hours : x * usual speed = (x + 25/60) x 6/7 usual speed => x = 6x/7 + 5/14 => x/7 = 5/14 :. x = 5/14 x 7 = 2 hours 30 minutes 20. Walking at 5 km/hr a student reaches his school from his house 15 minutes early and walking at 3 km/hr he is late by 9 minutes. What is the distance between his school and his house ? (a) 5 km (b) 8 km (c) 3 km - Answer (d) 2 km Expl : 5 x 3 x (15 + 9) = 15 x 24 = 3 km (5-3) 60 2 60 Directions : Q.21-23 Which of the following is most similar in meaning to the word given in bold as used in the passage. 21. Make (a) estimate (b) prepare (c) build (d) settle (e) earn - Answer 22. lost (a) defeated (b) failed (c) forfeited (d) wasted - Answer (e) destroyed 23. getaway (a) holiday (b) freedom (c) fantasy (d) escape - Answer (e) relaxation Directions-(Q.24-26) Read each sentence to find out whether there is any grammatical error in it. The error, if any, will be in one part of the sentence. The letter of that part is the answer. If there is no error, the answer is (E) i.e. 'No error'. (Ignore the errors of punctuation, if any). 24. Mr. Gupta insisted (A)/ that he was (B)/ fully prepare (c) / to present the proposal. (D) No error(E) Answer- C 25. All this document (A)/ have been destroyed (B)/ in the fire (C) / which occurred last month. (D) No error (E) Answer : (A) 26. On his next (A) /official visit to Delhi (B)/ he plans to (C)/visit his old colleagues. (D) No error (E) Answer : (C) Directions - (Q.27-28) Pick out the most effective word/group of words from those given to fill in the blanks to make the sentence meaningfully complete. 27. Although he appears to be very------in his speech, he is very good at heart. (a) Strong (b) fine (c) rude - Answer (d) innocent (e) talkative 28. He was wrongly punished although he was not-------- (a) fault (b) mistaken (c) erroneous (d) guilty (e) accused - Answer Directions - (Q.29-30) Rearrange the following six sentence (a), (b), (C),, (d), (e) and (f) in the proper sequence to form a meaningful paragraph; then answer the questions given below them. (A) A few minutes later he was hauled back on board the ship, on the captain's orders. (B) One of the crew began to complain and moan about the situation and no one could silence him (C) Marvelling at the change, the passengers asked for an explanation, to which the captain replied, "We never appreciate how well off we are in a situation until we fall into a worse one". (D) During a voyage a great storm arose. (E) In exasperation the captain ordered him to be thrown overboard. (F) To everyone's surprise, the man immediately went back to his post and silently began to work. 29. Which of the following should be the 3rd sentence after rearrangement (a) (A) (b) (B) (c) (C) (d) (D) (e) (E) - Answer 30. Which of the following should be the 1st sentence after rearrangement (a) (A) (b) (B) (c) (C) (d) (D) - Answer (e) (E) UPSC IES mechanical UPSC IES mechanical Enng and electrical and electronics communication engg questions with answers,UPSC Indian Engineering Services (IES).Examination syllabus and Details,UPSC IES Civil Engineering,UPSC IES Mechanical Engineering,UPSC IES,Electrical Engineering and Electronics & Telecommunication Engineering.questions with answers for practice,UPSC examination question papers,UPSC General studies and General English questions with answers,ectromagnetic Theory; Electrical Materials; Electrical Circuits; Measurement & Instrumentation; ControlIES mechanical engg questions with answers 1. The depth of critical N.A. of the beam is (m.d) / r (m + r) / d (m x d) / (m + r) none of these The answer is (m x d) / (m + r) . 2. Minimum spacing between horizontal parallel reinforcement of the same size should not be less than _____ diameters 4 3 2 1 The answer is 2 . 3. The maximum span to depth ratio of a simply supported slab spanning in both direction is _____ 25 30 35 50 .The answer is 35 4. The purpose of ribbed slab is to provide Thermal insulation Acoustic insulation A plain ceiling All of the above .The answer is All of the above 5. In rectangular water tanks, maximum B.M. occurs when depth of water in the tank is equal to _____ the length of short wall 1/4 1/2 3/4 1/5 .The answer is 1/2 6. Rapid hardening cement is actually early _____ cement setting strength hardening curing .The answer is setting 7. The relation between tread I and rise R in cms is given by 4 + 2t = 30 2t + r = 30 2r + t = 60 2r + t = 30 .The answer is 2r + t = 60 8. Ductility of steel decreases with a percentage _____ of carbon steel decrease increase improvement reduction .The answer is increase 9. _____ ratio of steel, within elastic limit, varies from 0.25 to 0.33 Compression Extension Impact Poisson's The answer is Poisson's . 10. If unsupported length of a stanchion is 5 m, slenderness ratio is 100, the least radius of gyration of the cross-section is _____ cm 50 60 10 5 The answer is 5 . 11. The process of adding extra carbon to the surface of steel is known as _____ Heat treatement Carbon coating Carburising Carbon treatement The answer is Carbon treatement. 12. Hardening of metal is due to heating to a particular temperature and followed by rapid cooling in a _____ media Cooling Tempering Quenching Freezing The answer is Quenching . 13. Maximum shear force occurs at the edge of a base plate in a _____ footing Stepped Rectangular Square Grillage .The answer is Rectangular 14. The slenderness ratio of a tension member should not exceed 350, if the member is subjected to _____ stresses resulting from wind force Compressive Tensile Bending Reversible .The answer is Reversible 15. 0.67 is the effective length of a compression member _____ Fixed at both ends Free at both ends Held in position and restrain in direction at both ends Fixed at both lower end and free at upper end .The answer is Held in position and restrain in direction at both ends 16. L/2 is the effective length of a column, fixed in _____ at its top and bottom position direction position & direction none of these .The answer is position & direction 17. 27t is the strength of ISA 155 x 75 x 10 used as a tie member with the longer leg connected at the ends by a _____ mm dia rivet 22 25 27 32 The answer is 27 . 18. The maximum deflection allowed in steel ties is L/750 L/480 L/350 unlimited The answer is L/350 . 19. The angle of inclination of lacing bars in steel ties should be more than 25 40 60 75 .The answer is 25 20. For normal loading conditions on a simply supported beam the breadth of the beam can be assumed as nearly 1/12 of span of the beam 1/16 of span of the beam 1/24 of span of the beam 1/30 of span of the beam .The answer is 1/30 of span of the beam 21. When a beam section spans over several supports continuously then the beam at supports should be designed as Singly reinforced beam Doubly reinforced beam Pre-stressed beam Pre-cast beam The answer is Doubly reinforced beam . 22. In a continuous slab the B.M. at middle of interior spans due to super imposed load of intensity W Kg/m run will be Wl^2 / 12 Wl^2 / 10 Wl^2 / 24 Wl^2 / 9 .The answer is Wl^2 / 12 23. As per IS : 456 in a two-way slab, the width of the middle strip along each span should be 3/8 of the width of span in that direction 3/8 of the width of span at right angles 3/4 of the shortest side of the slab 3/4 of width of the span in that direction The answer is 3/4 of width of the span in that direction . 24. The reinforcement provided in the form of ties or stirrups or binders in a column is known as longitudinal reinforcement transverse reinforcement temperature reinforcement distributary reinforcement .The answer is transverse reinforcement 25. Load carrying capacity of long columns are less as compared to short columns due to buckling effect of columns crippling effects of columns bending effects of columns twisting effect of columns The answer is buckling effect of columns . 26. As per IS : 456 the thickness of footing edge on soils should not be less than 10 cm 12 cm 15 cm 20 cm .The answer is 15 cm 27. Generally for piles the concrete used is of the grade M100 M150 M200 M300 The answer is M200 . 28. The shear force on a beam and the displacement are related by V = El. d^2 y / dx^2 V = El. d^3 y / dx^3 V = El. d^4 y / dx^4 no relation at all The answer is V = El. d^3 y / dx^3 29. The shear stress in a beam is zero At the centroid of the section On the extreme free surface fibers At the neutral axis but not at the centroid At the free edges .The answer is On the extreme free surface fibers 30. The minimum strength of concrete that can be used in the foundations of RCC water tanks of 450 mm thick is about M15 M20 M25 M10 .The answer is M15 31. The carbon content in steel does not exceed 0.5% 1.0% 1.5% 2.0% The answer is 2.0% 32 A streamline and an equipotential line in a flow field (A) Are parallel to each other (B) Are perpendicular to each other (C) Intersect at an acute angle (D) Are identical Answer: - (B) Explanation:- dy/dx* dy/dx=-1 Slope of equipotential Line × slope of stream function= -1 They are orthogonal to each line other. 33 If a mass of moist air in an airtight vessel is heated to a higher temperature, then (A) Specific humidity of the air increases (B) Specific humidity of the air decreases (C) Relative humidity of the air increases (D) Relative humidity of the air decreases Answer: - (D) Explanation:- R.H. Decreases Image 34 . In a condenser of a power plant, the steam condenses at a temperature of 0 60 C . The cooling water enters at 0 0 C and leaves at 0 45 C . The logarithmic ean temperature difference (LMTD) of the condenser is (A) 016.2 C (B) 0 21.6 C (C) 030 C (D) 0 37.5 C Answer: - (B) Explanation: - Flow configuration in condenser as shown below. 35 . The maximum possible draft in cold rolling of sheet increases with the (A) Increase in coefficient of friction (B) Decrease in coefficient of friction (C) Decrease in roll radius (D) Increase in roll velocity Answer: - (A) 36 . The operation in which oil is permeated into the pores of a powder metallurgy product is known as (A) Mixing (B) Sintering (C) Impregnation (D) Infiltration Answer: - (C) 37 . A hole is dimension 0.01509++f mm. The corresponding shaft is of dimension 0.0100.0019++ f mm. The resulting assembly has (A) Loose running fit (B) Close running fit (C) Transition fit D) Interference fit Answer: - (C) P Q P Q 1 kN m- 1m 3 38 . Heat and work are (A) Intensive properties (B) Extensive properties (C) Point functions (D) Path functions Answer: - (D) Explanation: - Heat and work are path functions. Since dQ and dW are dependent on path followed between two given end states of a thermodynamic process undergone by system. 39 . A column has a rectangular cross-section of 10mm x 20mm and a length of 1m. The slenderness ratio of the column is close to (A) 200 (B) 346 (C) 477 (D) 1000 Answer: - (B) I 40 . Green sand mould indicates that (A) Polymeric mould has been cured (B) Mould has been totally dried (C) Mould is green in colour (D) Mould contains moisture Answer: - (D) UPSC Numerical Aptitude UPSC IES/ISS General ability, Aptitude questions with answers and explanation, UPSC previous years solved questions with answers Part - C UPSC questions with answers and explanations Numerical Aptitude 101. If p = 124, 3vp (p2 + 3p + 3) + 1 = ?a. 5 b. 7 c. 123 d. 125 (Ans) Ans : 3vp (p2 + 3p + 3) + 1 = 3vp3 + 3p2 + 3p + 1 = 3v(p + 1)3= (p + 1) = 125 102. If v1 - x3/100 = 3/5, then x equals — a. 2 b. 4 (Ans) c. 16 d. (136)1/3 Ans : v1 - x3/100 = 3/5? 1 - x3/100 = 9/25? x3/100 = 16/25? x3 = 16*100/25 = (4)3? x = 4 103. I multiplied a natural number by 18 and another by 21 and added the products. Which one of the following could be the sum?a. 2007 (Ans) b. 2008 c. 2006 d. 2002 Ans : 18x + 21y = 3 (6x + 7y)Only 2007 is divisible by 3. Hence it is only possible. 104. The product of two numbers is 45 and their difference is 4. The sum of squares of the two numbers is —a. 135 b. 240 c. 73 d. 106 (Ans)Ans : x y = 45 x - y = 4? x2 + y2 = (x - y)2 + 2xy = 16 + 90 = 106 105. v8 + v57 + v38 + v108 + v169 = ?a. 4 (Ans) b. 6 c. 8 d. 10 Ans : ? = v8 + v57 + v 38 +v108 + v 169 = v 8 + v 57 + v38 + v108 +13 = v 8 + v57 + 7 = v 8 + 8 = 4 106. The square root of 14 + 6v5 is — a. 2 + v5 b. 3 + v5 (Ans) c. 5 + v3 d. 3 + 2v5 Ans : v 14 + 6 v 5 = v9 + 5 + 2*3* v5= 3 + v5 107. When 231 is divided by 5 the remainder is — a. 4 b. 3 (Ans) c. 2 d. 1 Ans : The unit digit in (2)31 will be same as the unit digit in (2)3, because the last digit is repeated after each 4 index? (2)31 ? 228 + 3 = 23 = 8? After dividing by 5, the remainder will be = 8 - 5 = 3. 108. The value of 1 + 1 is 1 + 1 1 + 1 1 + 1 1 + 2/3a. 21/13 b. 17/3 c. 34/21 (Ans) d. 8/5 Ans : 1 + 1 is 1 + 1 1 + 1 1 + 1 1 + 2 /3 = 1 + 1 1 + 1 1 + 1 1 + 3/5 = 1 + 1 1 + 1 1 + 5/8 = 1 + 1 1 + 8/13 = 1 + 13/21 = 34/21109. The unit digit in the product (122)173 is — a. 2 (Ans) b. 4 c. 6 d. 8 Ans : A 110. The value of 2 + v3 / 2 - v3 + 2 - v3 / 2 + v3 + v3 + 1 / v3 - 1 is—a. 16 + v3 (Ans)b. 4 - v3c. 2 - v3d. 2 + v3 Ans : 2 + v 3 / 2 - v3 + 2 - v3 / 2 + v3 + v3 + 1 / v3 - 1= (2 + v3)2 + (2 - v3)2 / ( 2 - v3) + (2 + v3) + (v3 + 1) (v3 + 1) / (v3 - 1) (v3 + 1)= 14/1 + 3 + 1 + 2v3/2 = 14 + 2 + v3 = 16 + v3 111. If a*b = 2a + 3b - ab, then the value of (3*5 + 5*3) is—a. 10 (Ans) b. 6 c. 4 d. 2 Ans : a * b = 2a + 3b - ab? (3 * 5 + 5 * 3) = (2 * 3 + 3 * 5 - 3 * 5) + (2 * 5 + 3 * 3 - 5 * 3)= (6 + 15 - 15) + (10 + 9 - 15) = 6 + 4 = 10 112. Simplify : 0.0347 * 0.0347 * 0.0347 + (0.9653)3 / (0.0347)2 - (0.347) (0.09653) + (0.9653)2a. 0.9306 b. 1.0009 c. 1.0050 d. 1 (Ans) (Ans) Ans : 0.0347 * 0.0347 * 0.0347 + (0.9653)3 / (0.0347)2 - (0.347) (0.09653) + (0.9653)2= (0.0347)3 + (0.9653)3 / (0.0347)2 - (0.0347) (0.9653) + (0.9653)2 (0.0347 + 0. 9653)= [(0.0347)2 - (0.0347 * 0.9653) + (0.9653)2] / [(0.0347)2 - 0.0347 * 0.9653 + (0.9653)2] = 1 113. A copper wire is bent in the form of an equilateral triangle and has area 121 v3 cm2, If the same wire is bent into the form of a circle, the area (in cm2) enclosed by the wire is ( Take = 22/7)— a. 364.5 b. 693.5 c. 346.5 (Ans) d. 639.5 Ans : Area of equilateral ? = v3/ 4 (side)2? 121 v3 = v3/4 (side)2? (side) = v4 *121 = 22 cm? Length of wire = 3 * 22 = 66cm? 2 * 22/7 * r = 66r = 66 * 7 / 2 * 22 = 21 / 2 cm ? Area of the circle = 22/7 * 21/2 * 21/2= 346.5 cm2 114. A child reshapes a cone made up of clay of height 24 cm and radius 6 cm into a sphere. The radius (in cm) of the sphere is—a. 6 (Ans) b. 12 c. 24 d. 48 Ans : 3/4 r3 = 1/3 (6)2 * 24? r3 = 216 = (6)3? r = 6 cm 115. Water flows into a tank which is 200 m long and 150 m wide, through a pipe of crosssection 0.3 m * 0.2 m at 20 km/hour. Then the time (in hours) for the water level in the tank to reach 8 m is— a. 50 b. 120 c. 150 d. 200 (Ans) Ans : Reqd. time = 200 * 150 * 8/ 0.3 * 0.2 * 20000 hrs.= 200 hrs. 116. The product of two numbers is 2028 and their H.C.F. is 13. The number of such pairs is— a. 1 b. 2 (Ans) c. 3 d. 4 Ans : Let the numbers be 13x and 13y.? 13x * 13y = 2028 ? xy = 12? x = 3 and y = 4or x = 1 and y = 12? The number of pairs are = 2 117. Two equal vessels are filled with the mixtures of water and milk in the ratio of 3 : 4 and 5 : 3 respectively. If the mixtures are poured into a third vessel, the ratio of water and milk in the third vessel will be—a. 15 : 12b. 53 : 59c. 20 : 9d. 59 : 53 (Ans)Ans : Reqd. ratio = (3/7 + 5/8) / (4/7 + 3/8)= (24 + 35)/ (32 + 21) = 59 : 53 118. I am three times as old as my son. 15 years hence, I will be twice as old as my son. The sum of our ages is— a. 48 years b. 60 years (Ans) c. 64 years d. 72 years Ans : Let the present age of the son be x years? My present age = 3x years? 3x + 15 / x + 15 = 2/1 ? Reqd. sum = 45 + 15 = 60 years 119. Three bells ring simultaneously at 11 a.m. They ring at regular intervals of 20 minutes, 30 minutes, 40 minutes respectively. The time when all the three ring together next is—a. 2 p.m. b. 1 p.m. (Ans) c. 1.15 p.m. d. 1.30 p.m. Ans : L.C.M. of 20, 30 and 40 minutes= 120 minutes= 2 hours. ? Reqd. time = 1 p.m. 120. A and B together can do a work in 12 days. B and C together do it in 15 days, If A's efficiency is twice that of C, then the days required for B alone of finish the work is— a. 60 b. 30 c. 20 (Ans) d. 15 Ans : Let the time taken by A to finish the work be x days? C will take 2x days to finish the work ? Work of B for 1 day = 1/12 - 1/x and work of B for 1 day = 1/15 - 1/2x? 1/12 -1/x = 1/15 - 1/2x ? 1/2x = 1/12 - 1/15 = 1/60 ? x = 30 ? Work of B for 1 day = 1/12 - 1/30= 1/20 ? B alone will finsh the work in 20 days. 121. A and B can do a work in 12 days, B and C can do the same work in 15 days, C and A can do the same work in 20 days. The time taken by A, B and C to do the same work is—a. 5 days b. 10 days (Ans) c. 15 days d. 20 days Ans : Reqd. time= 12 * 15 * 20 * 2/ (12 * 15 + 15 * 20 + 12 * 20) = 7200/(180 + 300 + 240) = 10 days 122. A is 50% as efficient as B . C does half of the work done by A and B together. If C alone does the work in 20 days, then A, B and C together can do the work in—a. 5 2/3 days b. 6 2/3 days (Ans) c. 6 days d. 7 days Ans : Let the time taken by B to complete the work be x days? Time ,, ,, by A ,, ,,= 2x days ? Work done by by C for 1 day = 1/2 (1/x + 1/2x) = 3/4x ? 3/4x = 1/20? x = 15? Work done by (A + B + C) for 1 day = 1/30 + 1/15 + 1/20 = 9/60= 3/20 ? Reqd. time = 6 2/3 days 123. The ratio of the volumes of water and glycerine in 240 cc of mixture is 1 : 3. The quantity of water (in cc) that should be added to the mixture so that the new ratio of added to the mixture so that the new ratio of the volumes of water and glycerine becomes 2 :3 is—a. 55 b. 60 (Ans) c. 62.5 d. 64 Ans : The quantity of water to be added= 240 (3-2)/ (1 + 3) = 60 c.c. 124. At present, the ratio of the ages of Maya and Chhaya is 6 : 5 and fifteen years from now, the ratio will get changed to 9 : 8 Maya's present age is—a. 21 years b. 24 years c. 30 years (Ans) d. 40 years Ans : Let the present ages of Maya and Chhaya be 6x and 5x years respectively.6x + 15/ 5x + 15 = 9/8 48x + 120 = 45x + 135 x = 5 ? Present age of Maya is = 30 years. 125. The ratio of the income to the expenditure of a family is 10 : 7. If the family's expenses are Rs.10,500, then savings of the family is—a. Rs.4,500 (Ans) b. Rs.10,000 c. Rs.4,000 d. Rs.5,000 Ans : Let the income and expenditures be Rs.10x and Rs.7x respectively? 7x = 10500? x = Rs.1500? Savings = (10-7) * 1500= Rs.4500 126. The average mathematics marks of two Sections A and B of Class IX in the annual examination is 74. The average marks of Section A is 77.5 and that of Section B is 70. The ratio of the number of students of Section A and B is—a. 7 : 8 b. 7 : 5 c. 8 : 7 (Ans) d. 8 : 5 Ans : Let the numbers of A and B be n and P respectively? n * 77.5 + P * 70 = (n + P) * 7477.5n - 74n = 74P - 70P 3.5n = 4P ? n/P = 4 : 3.5= 8 : 7 127. The average weight of a group of 20 boys was calculated to be 89.4 kg and it was later discovered that one weight was misread as 78 kg instead of 87 kg. The correct average weight is—a. 88.95 kg b. 89.25 kg c. 89.55 kg d. 89.85 kg (Ans) Ans : The correct average wt. = 20 * 89.4 + 87 - 78/ 20 = 1788 +9/20 = 89.85 kg. 128. The diameter of a wheel is 98 cm. The number of revolutions in which it will have to cover a distance of 1540 m is— a. 500 (Ans) b. 600 c. 700 d. 800 Ans : No. of revolutions= 1540 * 100 * 7/98 * 22 = 500 129. In an equilateral triangle ABC of side 10cm, the side BC is trisected at D. Then the length (in cm) of AD is— a. 3v7 b. 7v3 c. 10v7/3 (Ans) d. 7v10/3Ans : cos 60 = (10)2 + (10/3)2 - AD2/ 2 * 10 * 10/3 1/2 = 100 + 100/9 - AD2 / 200/31/2 * 200/3 = 1000 - 9 AD2 /9 AD2 = 700/9 ? AD = 10v7/3 cm 130. The cost price of an article is Rs.800. After allowing a discount of 10% , a gain of 12.5% was made. Then the marked price of the article is—a. Rs.1,000 (Ans) b. Rs.1,100 c. Rs.1,200 d. Rs.1,300 Ans : S. P. of the article = 800 * 112.5/100= Rs.900 ? M.P. of the article = 100 * 900/ (100 - 10) = Rs.1000 131. A man bought an article listed at Rs.1,500 with a discount of 20% offered on the list price. What additional discount must be offered to the man to bring the net price to Rs.1,104 ?a. 8% (Ans) b. 10% c. 12%d. 15% Ans : S. P. the article after a discount of 20% = 1500 * 80/100 = Rs.1200 ? Additional discount = (1200 - 1104) * 100/1200= 8% 132. If a/b = c/d = e/f = 3, then 2a2 + 3c2 + 4e2/ 2b2 + 3d2 + 4f2 = ?a. 2 b. 3 c. 4 d. 9 (Ans) Ans : a/b = c/d = e/f = 3? a = 3bc = 3d and e = 3f? ? = 2a2 + 3c2 + 4e2/ 2b2 +3d2 + 4f2= 2 * 9b2 + 3 * 9d2 + 4 * 9f2/ 2b2 + 3d2 + 4f2 = 9 133. The floor of a room is of size 4 m * 3m and its height is 3 m. The walls and ceiling of the room require painting. The area to be painted is— a. 66 m2 b. 54 m2 (Ans) c. 43 m2 d. 33 m2 Ans : Reqd. area = 2(4 + 3) * 3 + 4 * 3 = 42 + 12 = 54 m2 134. When the price of an article was reduced by 20%, its sale increased by 80%. What was the net effect on the sale? a. 44% increase (Ans) b. 44% decrease c. 66% increase d. 75% increase Ans : Reqd. % effect = [80 - 20 - 80*20/100]%= 44% increase 135. The price of sugar goes up by 20%. If a housewife wants the expenses on sugar to remain the same, she should reduce the consumption by— a. 15 1/5%b. 16 2/3% (Ans)c. 20% d. 25% Ans : Reqd. % reduction =20 * 100/ (100 + 20)% = 16 2/3% 136. In a factory 60% of the workers are above 30 years and of these 75% are males and the rest are females. If there are 1350 male workers above 30 years, the total number of workers in the factory is—a. 3000 (Ans) b. 2000 c. 1800 d. 1500 Ans : Let the total no.of workers be x. ? No. of males above 30 years ? x * 60/100 * 75/100 = 1350 ? x = 1350 * 100 * 100/ 60 * 75= 3000 137. Walking at 3/4 of his usual speed, a man is 1 1/2 hours late. His usual time to cover the same distance, in hours is—a. 4 1/2 (Ans) b. 4 c. 5 1/2 d. 5 Ans : Let the usual time be x hours x * usual speed= 3/4 * usual speed * (x + 3/2) x = 3/4 x + 9/8 ? x = 9/8 * 4= 4 1/2 hours 138. The selling price of 10 oranges is the cost price of 13 oranges. Then the profit percentages is—a. 30% (Ans) b.10% c. 13% d. 3% Ans : Profit % = (13-10)/10 * 100% = 30% 139. The marked price of a radio is Rs.480. The shopkeeper allows a discount of 10% and gains 8%. If no discount is allowed, his gain percent would be—a. 18% b. 18.5% c. 20.5% d. 20% (Ans) Ans : C.P. of the radio = 480 * 90/ 100 * 100/108= Rs.400 ? New profit % = 480 - 400/400 * 100%=20% 140. A man sold 20 apples for Rs.100 and gained 20%. How many apples did he buy for Rs.100?a. 20 b. 22 c. 24 (Ans) d. 25 Ans : ? C.P. of 1 apple = Rs. 100/20 * 100/120? No. of reqd. apples = 100 * 20 * 120/ 100 * 100= 24 141. A rectangular sheet of metal is 40 cm by 15 cm. Equal squares of side 4 cm are cut off at the corners and the remainder is folded up to form an open rectangular box. The volume of the box is—a. 896 cm3 (Ans) b. 986 cm3 c. 600 cm3 d. 916 cm3 Ans : Vol. of the open box = (40 - 8) * (15 - 8) * 4 = 32 * 7 * 4 = 896 cm3 142. If 78 is divided into three parts which are in the ratio 1 : 1/3 : 1/6, the middle part is—a. 9 1/3 b. 13 c. 17 1/3 (Ans) d. 18 1/3 Ans : Ratio = 1 : 1/3 : 1/6= 6 : 2 : 1 ? The middle part = 2 * 78/(6 + 2 + 1)= 52/3 = 17 1/3 143. The simple interest on a sum of money is 1/9 of the principal and the number of years is equal to rate per cent per annum. The rate per annum is—a. 3% b. 1/3% c. 3 1/3% (Ans) d. 3/10% Ans : ? 1/9 * P = P * R * R/100? R = 10/3%= 3 1/3% 144. The difference between simple interest and compound interest of a certain sum of money at 20% per annum for 2 years is Rs.48. Then the sum is—a. Rs.1,000 b. Rs.1,200 (Ans) c. Rs.1,500 d. Rs.2,000 Ans : ? D = P (r/100)2? 48 = P (20/100)2 ? P = 48 * 25= Rs.1200 145. Shri X goes to his office by scooter at a speed of 30 km/h and reaches 6 minutes earlier. If he goes at a speed of 24 km/h, he reaches 5 minutes late. The distance to his office is—a. 20 km b. 21 km c. 22 km (Ans) d. 24 km Ans : Reqd. distance = 30 * 24/(30 - 24) * (6 + 5)/60= 720 * 11/ 6 * 60 = 22 km 146. A sum of money becomes eight times in 3 years, if the rate is compounded annually. In how much time will the same amount at the same compound rate become sixteen times?a. 6 years b. 4 years (Ans) c. 8 years d. 5 years Ans : Reqd. time = a log q/ log p= 3 * log 16/ log 8 = 3 * 4 log 2/ 3 log 2 = 4 years Directions - The pie chart given below shows the spending of a family on various heads during a month. Study the graph and answer questions 147 to 150.(Image) 147. If the total income of the family is Rs.25,000, then the amount spent on Rent and Food together is—a. Rs.17,250 b. Rs.14,750 (Ans) c. Rs.11,250 d. Rs.8,500 Ans : Reqd. expenditure = (45 + 14) * 25000/100 = Rs.14750 148. What is the ratio of the expenses on Education to the expenses on Food? a. 1 : 3 (Ans) b. 3 : 1 c. 3 : 5 d. 5 : 3 Ans : Reqd. ratio = 15 : 45 = 1 : 3 149. Expenditure on Rent is what per cent of expenditure on Fuel ? a. 135% b. 156% (Ans) c. 167% d. 172% Ans : Reqd. ratio % = 14 * 100/9%= 156% 150. Which three expenditures together have a central angle of 1080 ?a. Fuel, Clothing and Others b. Fuel, Education and Others (Ans) c. Clothing, Rent and Others d. Education, Rent and others UPSC IAS,SSC,Assisst Commands General Science 1. Arrange the following electromagnetic waves in the descending order of their wavelength in the Electromagnetic Spectrum: 1.Infra-Red 2.Ultra-Violet 3.Microwaves 4.Radiowaves [A]4-3-1-2-Answer [B]1-2-3-4 [C]2-3-1-4 [D]2-1-3-4 2. Television Remote Controls and Night-Vision Goggles are operated using : [A]Microwaves [B]Infra-Red rays [C]Ultra-Violet Rays [D]LASER Answer-Infra-Red rays 3. GPRS is the abbreviated form of: [A]General Packet Remote Synchronization [B]Globally Postioned Radio System [C]General Packet Radio Service [D]Globally Positioned Radio Service Answer-General Packet Radio Service 4. Blue-tooth networking transmits data via low power _______ : [A]Microwaves [B]Radiowaves [C]Infrared Rays [D]Gamma Rays Answer-Radiowaves 5. After alcohol consumption person cannot walk properly because alcohol affects his: [A]Spinal Cord [B]Cerebrum [C]Cerebellum [D]Medulla Oblongata Answer-Cerebellum 6. 'Cretinism' is caused by the under-secretion of which of the following hormones? [A]Adrenaline [B]Thyroxin [C]Testosterone [D]Insulin Answer-Thyroxin 7. Consider the following plants: 1.Jatropha 2.Eucalyptus 3.Pongamia Which among the above are used to obtain 'Biodiesel'? [A]Only 1 [B]1 and 3 [C]2 and 3 [D]1, 2 and 3 Answer-1 and 3 8. Astigmatism is a _____ disorder: [A]Visual [B]Olfactory [C]Genetic [D]Hormonal Answer-Visual 9. Consider the following Statements regarding 'Chlorophyll' pigment: 1.It is a poor absorber of green and near-green wavelengths of the electromagnetic spectrum. 2.Chlorophyll molecule complex centred around a Magnesium ion. Which of the Statement(s) given above are correct? [A]Only 1 [B]Only 2 [C]Both 1 and 2 [D]Neither 1 nor 2 Answer-Both 1 and 2 10. 'Histology' is a branch of biology which deals with the study of : [A]Tissues [B]Cell division [C]Prokaryotes [D]Ageing and Senescence Answer-Tissues 11. Which among the following is the smallest gland of the human body? [A]Pineal Gland [B]Islets of Langerhans [C]Adrenal Cortex [D]Thyroid Gland Answer-Pineal Gland 12. Which of the following is the 'least distance of distinct vision' for a healthy human eye? [A]25 cm [B]45 cm [C]50 cm [D]90 cm Answer-25 cm 13. Consider the following properties of radioactive 'Alpha-Rays': 1.Alpha Rays consist of doubly-ionised Helium atoms. 2.They have the highest ionising and penetrating power. Which of the above Statement(s) is/are correct? [A]Only 1 [B]Only 2 [C]Both 1 and 2 [D]Neither 1 nor 2 Answer-Only 1 Alpha Rays have the highest Ionization power but Gamma Rays have the highest penetrating power. 14. Which among the following chemical compounds is used to bring 'artificial rain'? [A]Magnesium Chloride [B]Silver Iodide [C]Potassium Bromide [D]Sodium Hypochlorite Answer-Silver Iodide 15. Which among the following is being used as fuel in the 'Fast-Breeder' Reactor of Kalpakkam? [A]Polonium [B]Thorium [C]Plutonium [D]Enriched-Uranium Answer-Thorium 16. Which among the following is the edible part in Rice? [A]Fruit [B]Cotyledons [C]Endosperm [D]Embryo Endosperm It?s a very common question but can confuse many of you. Most of the essential micronutrients are almost exclusively stored in the husk, aleurone and embryo of rice, which are removed during the polishing process. Polishing of rice is required as the oil-rich aleurone layer turns the seed rancid upon storage and therefore making rice unsuitable for consumption. Consequently, the rice endosperm that comprises the edible part of rice for humans lacks or contains only small amount of key micronutrients (e.g., iron, zinc, protein, provitamin A, and other vitamins) that are essential for a healthy diet. Considering the facts above and severity of widespread micronutrient malnutrition, iron biofortification in rice endosperm is a promising strategy to overcome iron deficiency effectively. 17 In context with computers, what can be the uses of cookies? 1. Maintain Information 2. Run a programme 3. Deliver a Virus Choose the correct option: [A]Only 1 [B]1 & 2 [C]2 & 3 [D]1, 2 & 3 Only 1 A cookie is a text string that is included with Hypertext Transfer Protocol requests and responses. Cookies are used to maintain information as you navigate different pages on a website or return to the website at a later time. Cookies cannot be used to run code (programmes) or deliver viruses to your computer. One of their primary purposes is to provide a convenience feature that you can use to save time ? it tells the web server that you have returned to the page. If you personalize web pages or register for products and services, a cookie helps the page server recall your specific information, like billing address, shipping address. 18. How does calcium carbide ripen mangoes? [A]It reacts with moisture and releases Ethylene [B]It reacts with moisture and releases Ethyne [C]It reacts with moisture and releases Acetaldehyde, which in turn releases ethylene [D]It reacts with starch and releases Acetylene It reacts with moisture and releases Ethyne For reasons of safe transportation of fruits, mangoes, bananas etc are picked before they ripen fully. Slightly green harvested mangoes are subjected to small containers of calcium carbide (CaC2) with a plastic covering. CaC2 reacts with the moisture in the air to release acetylene (or ethyne) gas as follows: CaC2 + 2 H2O ? C2H2 + Ca(OH)2 This option may give you slightest confusion if you don't know the difference between Ethylene and Ethyne (Acetylene). Both are chemical substances produced by fruits to accelerate the normal process of fruit maturation ? is a ripening hormone. Ethylene is C2H4 and has a carbon-carbon double bond. Acetylene is C2H2 and has a carbon-carbon triple bind. Acetylene contains more energy and burns hotter than ethylene. Ethylene is a gaseous compound , Acetylene is the chemical compound , which is the simplest alkyne and is also a hydrocarbon while Ethylene is the simplest alkene and classified as an unsaturated hydrocarbon. I hope you will never forget it now. 19 Consider the following statements about Cloud Computing: 1. It is a service that is generally offered by a third party 2. It includes only Software and not hardware 3. It is generally offered on one to one basis Which among the above statements is/ are correct? [A]1 & 2 [B]1 & 3 [C]Only 1 [D]1, 2 & 3 Only 1 Cloud computing with reference to IT is a service generally offered by a third party ? typically a Technology Services Provider ? and offered on subscription basis by leveraging the internet. The key proposition is to move away from expensive, capex heavy, on-premises options to affordable, cost- efficient arrangements that are offered on a one-to-many basis. It is a misperception that cloud computing refers to only software; it includes hardware as a service enabling computing on demand. 20. What are the basic premises of Software Entropy? 1. A computer program that is used will be modified 2. When a program is modified, its complexity will increase, provided that one does not actively work against this. 3. It may be better to redesign the software rather than build on the existing programme Choose the correct option: [A]Only 1 [B]1 & 2 [C]1, 2 & 3 [D]1 & 3 1, 2 & 3 Entropy means disorder. The tendency, over time, for software to become difficult and costly to maintain is known as software entropy. A software system that undergoes continuous change, like new functionality added to its original design, will eventually become more complex, losing its original design structure, and may even become disorganized as it grows. In theory, it may be better to redesign the software in order to support the changes rather than build on the existing programme. 21. If a person wants to increase the level of good Cholesterol in his / her body, which among the following you would advise? 1. Increase the use of trans fatty acids in diet 2. Stop smoking 3. Consumption of omega-3 fatty acids Choose the correct option: [A]1, 2 & 3 [B]Only 1 & 3 [C]Only 2 & 3 [D]Only 3 Only 2 & 3 Trans fats are not essential fatty acids. The consumption of trans fats increases the risk of coronary heart disease by raising levels of LDL cholesterol and lowering levels of "good" HDL cholesterol. 22. Why do the bacteria develop resistance after slowly being exposed to a particular antibiotic? 1. Due to mutation in genes 2. Due to Horizontal Gene Transfer 3. Due to Vertical Gene Transfer Choose the correct option: [A]Only 1 [B]Only 1 & 2 [C]Only 2 & 3 [D]1, 2 & 3 Only 1 & 2 This is very carefully framed question and would clear the concept of antibiotic resistance, Horizontal Gene Transfer and Vertical Gene Transfer. Antibiotic resistance occurs from random mutations, the first choice given is correct. The slow exposure to an antibiotic will give more chance for mutations to occur. Most antibiotics are derived from naturally derived from organisms, penicillin was derived from the mould Penicillium, other antibiotics have also been extracted from bacteria, and these bacteria will be resistant to that antibiotic. The Horizontal Gene Transfer is considered to be the main culprit of the antibiotic resistance. Horizontal gene transfer (HGT), also lateral gene transfer (LGT), is any process in which an organism incorporates genetic material from another organism without being the offspring of that organism. By contrast, vertical transfer occurs when an organism receives genetic material from its ancestor, e.g., its parent or a species from which it has evolved. Horizontal gene transfer is also the primary reason for bacterial antibiotic resistance and this often involves plasmids. Genes that are responsible for antibiotic resistance in one species of bacteria can be transferred to another species of bacteria through various mechanisms. The resistance spreads across bacterial species usually by transfer of genetic material via a vector, this vector tends to be a circular piece of genetic material known as a plasmid. Operons within the plasmid code for several genes, a combination of proteins including an enzyme which is able to deactivate antibiotics, for example beta-lactamase is an enzyme which cleaves beta-lactam rings, deactivating penicillin and its derivatives (although certain derivatives like methicillin are resistant to beta lactamases). Sequences within plasmids that can transfer to the cell's own genetic material is known as a transposon. The third choice "vertical gene transfer' is not correct. Vertical Gene Transfer means reproduction. 23 Which among the following higher mammals has been shown to hibernate? [A]Gorilla [B]Chimpanzee [C]Black Bear [D]Rhesus Monkey Black Bear 24. Which among the following is not among Micronutrients required for plants? [A]Molybdenum [B]Magnesium [C]Manganese [D]Zinc Magnesium In order for a plant to grow and thrive, it needs a number of different chemical elements. The most important are: Carbon, hydrogen and oxygen - Available from air and water and therefore in plentiful supply Nitrogen, phosphorus, potassium (a.k.a. potash) - The three macronutrients and the three elements you find in most packaged fertilizers Sulfur, calcium, and magnesium - Secondary nutrients Boron, cobalt, copper, iron, manganese, molybdenum and zinc - Micronutrients 25. What does the word "Synthia" refer to in context with a recent discovery / invention? [A]A plant cell [B]A red blood cell [C]A bacteria [D]A fungus A bacteria Synthia refers to the "Mycoplasma laboratorium". It was announced researchers had developed world's first living cell controlled by synthetic DNA. The organism nicknamed "Synthia" is controlled by what the group calls "genetic software". It was done at J. Craig Venter Institute by a team of approximately twenty scientists headed by Nobel laureate Hamilton Smith, and including DNA researcher Craig Venter and icrobiologist Clyde A. Hutchison III. 27.In 2006, it was reported that doctors are finding more and more people, who are having both type 1 & type 2 diabetes. What is the new term coined for this threat? (A)Hybrid Diabetes (B)Double Diabetes (C)Both of Them-Answer 28.When the body's immune system cells destroy the insulin producing beta cells of the pancreas is termed as ________? (A)Type 1 diabetes-Answer (B)type 2 diabetes (C)hybrid diabetes 29.A person develops resistance to the action of insulin in the body's cells , it is called as___________? (A)Type 1 Diabetes (B)Type 2 Diabetes-Answer (C)none of them 30 Athlete's Foot or Tinia Pedis is caused by which of the following ? (A)A Bacteria (B)A Virus (C)A Fungi-Answer 5.Leishmaniasis is a disease caused by protozoan parasites that belong to the genus Leishmania and is transmitted by the bite of certain species of_________? (A)Mosquito (B)Sand Fly-Answer (C)Bee 31.Out of Diphtheria, Whooping cough, Typhoid , Diarrhea & Malaria which is not caused by bacteria? (A)Whooping Cough (B)Typhoid (C)Malaria-Answer 32.The 2009 flu pandemic is a global outbreak of a new strain of influenza A virus subtype H1N1 which is a mutation of four known virus strains of influenza A virus subtype H1N1. This disease typically spreads by ___________? (A)Using Pork as Food (B)from coughs and sneezes or by touching contaminated surfaces and then touching the nose or mouth-Answer (C)Working with Pigs 33.Sickle cell anemia. Albinism. Color blindness, phenylketonuria, Klinefelter syndrome, Turner syndrome & Down syndrome are mainly grouped as ? (A)Infectious Diseases (B)Congenital Diseases-Answer (C)Organic Diseases 34.Schizophrenia is an example of _________? (A)Allergy (B)Mental Illness-Answer (C)Organic Degenerative Diseases 35.Hepatitis A which is the most common cause of jaundice in young people is an infection of liver by ________? (A)Bacteria (B)Virus-Answer (C)Amoeba 36.Poliomyelitis known as Polio is caused by an enterovirus which causes paralysis of muscles. This virus has a particular affinity to ______? (A)Muscle Cells (B)Nerve Cells of Spinal Cord-Answer (C)Nerve Cells of Brain 37.The WHO has declared the world free of which of the following disease? (A)Small Pox (B)Viral Pneumonia-Answer (C)Cholera 38.Meningitis is a disease which is characterized by inflammation of meninges , the covering that lie over the brain and spinal chord is caused by _______ ? (A)Bacteria-Answer (B)Virus (C)Protozoa 39.Which among the following disease is called by Yersinia pestis? (A)Leprosy (B)Anthrax (C)Plague-Answer 40.Dengue which is found in tropical and subtropical regions around the world is a mosquito borne disease and caused by a _________? (A)Bacteria (B)Protozoa (C)Virus-Answer 41.Enterobius vermicularis which affect the caecum and vermiform appendix is commonly called as ? (A)tapeworm-Answer (B)pinworm (C)threadworm 42Polycythaemia is a disease related to _______? (A)Red Blood Cells-Answer (B)White Blood Cells (C)Blood Serum 43.Gout which is usually followed / accompanied by arthritis are characterized by high levels of uric acid in the blood and uric acid crystals become lodged in the joints. It is a ________? (A)Viral Infection (B)Metabolic Disorder-Answer (C)Muscle Disorder 44.Down syndrome, Down's syndrome, or trisomy 21 is a chromosomal disorder caused by the presence of all or part of an extra 21st chromosome. What is other name of Down Syndrome? (A)Edwards syndrome (B)Patau syndrome-Answer (C)Mongolism 45.Diabetes insipidus (DI) caused by a deficiency of arginine vasopressin (AVP), also known as antidiuretic hormone (ADH). This hormone is secreted by which endocrine gland? (A)Thyroid Gland (B)Pituitary Gland (Hypophysis)-Answer (C)Pancreas UPSC current affairs questions with answers UPSC written test examination current affairs and general awareness questions with answers, UPSC previous years solved question papers, UPSC General ability, Economy, Intelligence reading comprehension and general studies questions with answers,UPSC Previous year solved Question paper with Answer, UPSC Solved Question paper for Assistant Commandants Exam 2012,UPSC model Question paper with Answer, UPSC Latest Assistant Commandants Exam details, UPSC Latest recruitment of Assistant Commandants 2012 latest Selection pattern, Written Exam details, UPSC Latest Selection procedure for Central Armed Police Force Exam 2012, UPSC General Ability and intelligence Questions with Answers, New UPSC Written Exam Syllabus for Assistant commandant Exam 2012, UPSC General studies Questions and comprehension Question pattern for recruitment of Assistant Commandant Exam 2012 1. Article 17 of the constitution of India provides for (a) equality before law. (b) equality of opportunity in matters of public employment. (c) abolition of titles. (d) abolition of untouchability-Answer 2. Article 370 of the constitution of India provides for (a) temporary provisions for Jammu & Kashmir.-Answer (b) special provisions in respect of Nagaland. (c) special provisions in respect of Manipur. (d) provisions in respect of financial emergency 3. How many permanent members are there in Security Council? (a) Three (b) Five-Answer (c) Six (d) Four 4. The United Kingdom is a classic example of a/an (a) aristocracy (b) absolute monarchy (c) constitutional monarchy-Answer (d) polity. 5. Social Contract Theory was advocated by (a) Hobbes, Locke and Rousseau.-Answer (b) Plato, Aristotle and Hegel. (c) Mill, Bentham and Plato. (d) Locke, Mill and Hegel. 6. The Speaker of the Lok Sabha is elected by the (a) President (b) Prime Minister. (c) Members of both Houses of the Parliament. (d) Members of the Lok Sabha.-Answer 7. Who is called the ‘Father of History'? (a) Plutarch (b) Herodotus-Answer (c) Justin (d) Pliny 8. The Vedas are known as (a) Smriti. (b) Sruti.-Answer (c) Jnana. (d) Siksha. 9. The members of Estimate Committee are (a) elected from the Lok Sabha only.-Answer (b) elected from the Rajya Sabha only. (c) elected from both the Lok Sabha and the Rajya Sabha. (d) nominated by the Speaker of the Lok Sabha. 10. Who is the chief advisor to the Governor? (a) Chief Justice of the Supreme Court. (b) Chief Minister.-Answer (c) Speaker of the Lok Sabha. (d) President. 11. Foreign currency which has a tendency of quick migration is called (a) Scarce currency. (b) Soft currency. (c) Gold currency. (d) Hot currency.-Answer 12. Which of the following is a better measurement of Economic Development? (a) GDP-Answer (b) Disposable income (c) NNP (d) Per capita income 13. In India, disguised unemployment is generally observed in (a) the agriculture sector.-Answer (b) the factory sector. (c) the service sector. (d) All these sectors. 14. If the commodities manufactured in Surat are sold in Mumbai or Delhi then it is (a) Territorial trade. (b) Internal trade.-Answer (c) International trade. (d) Free trade. 15. The famous slogan "GARIBI HATAO" (Remove Poverty) was launched during the (a) First Five-Year Plan (1951-56) (b) Third Five-Year Plan (1961-66) (c) Fourth Five-Year Plan (1969-74)-Answer (d) Fifth Five-Year Plan (1974-79) 16. Bank Rate refers to the interest rate at which (a) Commercial banks receive deposits from the public. (b) Central bank gives loans to Commercial banks.-Answer (c) Government loans are floated. (d) Commercial banks grant loans to their customers. 17. All the goods which are scare and limited in supply are called (a) Luxury goods. (b) Expensive goods. (c) Capital goods. (d) Economic goods.-Answer 18. The theory of monopolistic competition is developed by (a) E.H.Chamberlin-Answer (b) P.A.Samuelson (c) J.Robinson (d) A.Marshall 19. Smoke is formed due to (a) solid dispersed in gas.-Answer (b) solid dispersed in liquid. (c) gas dispersed in solid. (d) gas dispersed in gas. 20. Which of the following chemical is used in photography? (a) Aluminum hydroxide (b) Silver bromide-Answer (c) Potassium nitrate (d) Sodium chloride. 21. Gober gas (Biogas) mainly contains (a) Methane.-Answer (b) Ethane and butane. (c) propane and butane. (d) methane, ethane, propane and propylene. 22. Preparation of ‘Dalda or Vanaspati' ghee from vegetable oil utilises the following process (a) Hydrolysis (b) Oxidation-Answer (c) Hydrogenation (d) Ozonoloysis 23. Which colour is the complementary colour of yellow? (a) Blue (b) Green-Answer (c) Orange (d) Red 24. During washing of cloths, we use indigo due to its (a) better cleaning action.-Answer (b) proper pigmental composition. (c) high glorious nature. (d) very low cost. 25. Of the following Indian satellites, which one is intended for long distance telecommunication and for transmitting TV programmes? (a) INSAT-A-Answer (b) Aryabhata (c) Bhaskara (d) Rohini 26. What is the full form of ‘AM' regarding radio broadcasting? (a) Amplitude Movement (b) Anywhere Movement (c) Amplitude Matching (d) Amplitude Modulation.-Answer 27. Who is the author of Gandhi's favorite Bhajan Vaishnava jana to tene kahiye? (a) Purandar Das (b) Shyamal Bhatt (c) Narsi Mehta-Answer (d) Sant Gyaneshwar 28. Which one of the following is not a mosquito borne disease? (a) Dengu fever (b) Filariasis (c) Sleeping sickness (d) Malaria-Answer 29. What is the principal ore of aluminium? (a) Dolomite (b) Copper (c) Lignite (d) Bauxite-Answer 30. Which country is the facilitator for peace talks between the LTTE and the Sri Lankan Government? (a) The US-Answer (b) Norway (c) India (d) The UK 31. The highest body which approves the Five-Year Plan in India is the (a) Planning Commission (b) National Development Council (c) The Union Cabinet-Answer (d) Finance Ministry 32. Ceteris Paribus is Latin for (a) " all other things variable " (b) "other things increasing" (c) "other things being equal"-Answer (d) "all other things decreasing" 33. Who has been conferred the Dada Saheb Phalke Award (Ratna) for the year 2007? (a) Dev Anand (b) Rekha (c) Dilip Kumar-Answer (d) Shabana Azmi 34. Purchasing Power Parity theory is related with (a) Interest Rate.-Answer (b) Bank Rate. (c) Wage Rate. (d) Exchange Rate. 35. India's biggest enterprise today is (a) the Indian Railways. (b) the Indian Commercial Banking System. (c) the India Power Sector.-Answer (d) the India Telecommunication System. 36. The official agency responsible for estimating National Income in India is (a) Indian Statistical Institute. (b) Reserve Bank of India. (c) Central Statistical Organisation. (d) National Council for Applied Economics and Research.-Answer 37. Which of the following has the sole right of issuing currency (except one rupee coins and notes) in India? (a) The Governor of India (b) The Planning Commission (c) The State Bank of India (d) The Reserve Bank of India-Answer 38. In the budget figures of the Government of India the difference between total expenditure and total receipt is called. (a) Fiscal deficit-Answer (b) Budget deficit (c) Revenue deficit (d) Current deficit 39. Excise duty on a commodity is payable with reference to its (a) production.-Answer (b) production and sale. (c) Production and transportation. (d) Production, transportation and sale. 40. In the US, the President is elected by (a) The Senate. (b) Universal Adult Franchise.-Answer (c) The House of Representatives. (d) The Congress. 41. Fascism believes in (a) Peaceful change (b) Force (c) Tolerance (d) Basic Rights for the individual-Answer 42. Which is the most essential function of an entrepreneur? (a) Supervision-Answer (b) Management (c) Marketing (d) Risk bearing 43. Knowledge, technical skill, education ‘etc.' in economics, are regarded as (a) social-overhead capital. (b) human capital.-Answer (c) tangible physical capital. (d) working capital. 44. What is the range of Agni III, the long-range ballistic missile, test-fired by India recently? (a) 2,250 km-Answer (b) 3,500 km (c) 5,000 km (d) 1,000 km 45. Nathu Laa, a place where India-China border trade has been resumed after 44 years, is located on the Indian border in (a) Sikkim. (b) Arunachal Pradesh. (c) Himachal Pradesh-Answer (d) Jammu and Kashmir. 46. M. Damodaran is the (a) Chairman, Unit Trust of India. (b) Deputy Governor of Reserve Bank of India. (c) Chairman, Securities and Exchange Board of India.-Answer (d) Chairman, Life Insurance Corporation of India. 47. What is the name of the Light Combat Aircraft developed by India indigenously? (a) BrahMos (b) Chetak (c) Astra (d) Tejas-Answer 48. Who is the Prime Minister of Great Britain? (a) Tony Blair (b) Jack Straw (c) Robin Cook (d) Gordon Brown.-Answer 49. The 2010 World Cup Football Tournament will be held in (a) France. (b) China.-Answer (c) Germany. (d) South Africa. 50. Who is the present Chief Election Commissioner of India? (a) Navin Chawla-Answer (b) N.Gopalswamy (c) T.S.krishnamoorty (d) B.B.Tandon 51. The title of the book recently written by Jaswant Singh, former Minister of External Affair, is (a) A call of Honour - In the Service of Emergent Inida (b) Whither Secular India? (c) Ayodhya and Aftermath-Answer (d) Shining India and BJP. 52. What was the original name of "Nurjahan"? (a) Jabunnisa (b) Fatima Begum (c) Mehrunnisa (d) Jahanara-Answer 53. Which of the following pairs is not correctly matched ? (a) Lord Dallhousie- Doctrine of Lapse (b) Lord Minto- Indian Councils Act, 1909 (c) Lord Wellesley- Subsidiary Alliance (d) Lord Curzon- Vernacular Press Act, 1878-Answer 54. The province of Bengal was partitioned into two parts in 1905 by (a) Lord Lytton. (b) Lord Ripon.-Answer (c) Lord Dufferin. (d) Lord Curzon. 55. The essential features of the Indus Valley Civilization was (a) worship of forces of nature.-Answer (b) organized city life. (c) pastoral farming. (d) caste society. 56. Name the capital of Pallavas. (a) Kanchi. (b) Vattapi.-Answer (c) Trichnapalli. (d) Mahabalipuram. 57. The Home Rule League was started by (a) M.K.Gandhi (b) B.G.Tilak (c) Ranade-Answer (d) K.T.Telang 58. The Simon Commission was boycotted by the Indians because (a) it sought tocurb civil liberties. (b) it proposed to partition India. (c) it was an all-white commission Indian representation. (d) it proposed measures for nationalism.-Answer 59. Storm of gases are visible in the chamber of the Sun during (a) Cyclones (b) Anti-cyclonesAnswer (c) Lunar-eclipse- (d) Solar eclipse 60. The Indian Councils Act of 1990 is associated with (a) The Montagu Decleration.-Answer (b) The Montagu- Chelmsford Reforms. (c) The Morley-Minto Reforms. (d) The Rowlatt Act. 61. The age of tree can be determined more or less accurately by (a) counting the number of branches.-Answer (b) measuring the height ,of the tree. (c) measuring the diameter of the trunk. (d) counting the number of rings in the trunk. 62. Of all micro-organisms, the most adaptable and versatile are (a) Viruses-Answer (b) Bacteria (c) Algae d) Fungi 63. What is an endoscope? (a) It is an optical instrument used to see inside the alimentary canal-Answer (b) it is device which is fitted on the chest of the patient to regularize the irregular heart beats (c) It is an instrument used for examining ear disorders (d) It is an instrument for recording electrical signals produced by the human muscles. 64. The disease in which the sugar level increase is known as (a) Diabetes mellitus-Answer (b) Diabetes insipidus (c) Diabetes imperfectus (d) Diabetes sugarensis 65. The President of India is elected by (a) members of both Houses of the Parliament. (b) members of both houses of Parliament of State Legislatures. (c) members of both Houses of the State Legislative Assemblies. (d) Elected members of both Houses of the Parliament and members of Legislative Assemblies.-Answer 66. The nitrogen present in the atmosphere is (a) of no use to plants.-Answer (b) injurious of plants. (c) directly utilized by plants. (d) utilized through micro-organisms . 67. Diamond and Graphite are (a) allotropes (b) isomorphous-Answer (c) isomers (d) isobars 68. Kayak is kind of (a) tribal tool. (b) boat.-Answer (c) ship. (d) weapon. 69. Which of the following has the highest calorific value? (a) Carbohydrates-Answer (b) fats (c) Proteins (d) Vitamins. 70. Rotation of crops means (a) growing of different crops in succession to maintain soil fertility.-Answer (b) some crops are growing again and again. (c) two or more crops are grown simultaneously to increase productivity. (d) None of these. 71. Suez Canal connects (a) Pacific Ocean and Atlantic Ocean. (b) Mediterranean Sea and Red Sea.-Answer (c) Lake Huron and Lake Erie. (d) Lake Erie and Lake Ontario. 72. Which of the following ports has the largest hinterland? (a) Kandla (b) Kochi (c) Mumbai (d) Vishkhapatnam.-Answer 73. "Slash and Burn agriculture" is the name given to (a) method of potato cultivation.-Answer (b) process of deforestation. (c) mixed framing. (d) shifting cultivation. 74. The main reason for deforestation in Asia is (a) excessive fuel wood collection. (b) excessive soil erosion. (c) floods. (d) construction of roads.-Answer 75. Recharging of water table depends on (a) amount of rainfall.-Answer (b) relief of the area. (c) vegetation of the area. (d) amount of percolatio UPSC Genereal Awareness Section -2 (April-2012) 1. Name the freedom fighter with picture of whom Rs.5 coin would be introduced by RBI. Ans. Bhagat Singh 2. Name the news search engine introduced by Rediff.com Ans. Relatime News Search 3. Tiger brand is associated with which company? Ans. Britannia 4. Name the cricketer who was honoured with lifetime membership the Sydney Cricket Ground. Ans. Sachin Tendulkar 5. Which country has been ranked as the world’s richest country in the list compiled by US magazine Forbes? Ans. Qatar 6. Which country did India beat to qualify for London Olympics men’s hockey 2012? Ans. France 7. Name the service that is to be launched by Times Group to enable viewers to watch movies and TV shows online. Ans. BoxTV.com 8. Name the football player who led India to the gold in the inaugural Asian Games in New Delhi who passed away recently. Ans. Sailendra Nath Manna 9. Who won the Allan Border medal (Australia cricketer of the year) award 2012? Ans. Michael Clarke 10. Which west Asian country approved for new constitution recently? Ans. Syria 11. Which model of Nokia phone has been built with 41-megapixel sensor camera? Ans. Nokia 808 Pureview camera phone 12. Name the digital music store launched by flipkart.com. Ans. FlyteDownload From http://www.jbigdeal.com 13. Name the online property search portal launched by HDFC in Kolkata Ans. HDFC Red 14. Name the application launched by Vigyan Prasar and IGNOU as a free SMS service for mobile users that delivers content on science and related areas. Ans. Science@Mobile 15. Name the Kerala-based UAE business magnate who was awarded with Interfaith Harmony Award at Kuala Lumpur. Ans. M.A.Yusuf Ali 16. Which country has India agreed ‘in-principle’ to the participation in hydro power-projects in the North-East? Ans. Bangladesh 17. Name the Tech-savvy state which set a record by winning the National Tourism Award for its website for 5th time in last 10 years. Ans. Kerala 18. Who has been appointed as brand ambassador of fruit drink maker rasna? Ans. Genelia D’souza 19. Name the book which is set to be launched by model-turnedactress Diana Hayden. Ans. A Beautiful Truth 20. Who has been awarded the Pritzker Architecture Prize considered as Nobel prize in Architecture? Ans. Wang Shu 21. Which railway station became the first railway network to offer BluFi, a combination of Bluetooth and WiFi to provide information to passengers? Ans. Bangalore City Railway Station 22. Name the Indian golfer who qualified for The Open Championship to be played in July at the Royal Lytham St Annes. Ans. Anirban Lahiri 23. Which state bagged the second best national award in the Comprehensive Development of Tourism category? Ans. RajasthanDownload 24. Name the men’s mixed doubles winner of the ATP Dubai Open 2012. Ans. Mahesh Bhupathi and Rohan Bopanna 25. Who won the men’s singles of ATP Dubai Open 2012? Ans. Roger Federer 26. Name the Indian American filmmaker who won the ’2012 Rising Star Award’ for his feature film ’9 Eleven’ at the Canada International Film Festival. Ans. Manan Singh Katohora 27. Who has been elected as Russian President recently? Ans. Vladimir Putin 28. Name of Taslima Nasreen’s autobiography? Ans. Nirbasan 29. Who is World’s shortest man as per recent Guinness World Records ? Ans. 72-year old Chandra Bahadur Dangi of Nepal 30. Name the Romanian Prime Minister who quit recently. Ans. Emil Boc 31. Name the Maldives President who was appointed after Mohamed Nasheed quit. Ans. Mohammad Waheed Hassan 32. Which singer is launching her own social networking website exclusively for her fans? Ans. Lady Gaga [www.littlemonsters.com] 33. Which country released world’s highest resolution lunar images taken by its 2nd moon orbiter, Chang’e-2? Ans. China 34. Who was honoured with Lata Mangehkar award by the Madhya Pradesh government recently? Ans. Rajesh Roshan 35. Why zebras have black and white stripes (recently solved mystery)? Ans. To keep blood-sucking files away 36. Who was honoured with a posthumous Grammy award for his contributions to the music industry in area other than performance? Ans. Steve Jobs ---------------------------------------------------------------------------------------------------------------------------------------------------------------------------------------------------------- 1. Name the yesteryear actress who featured in the popular song ‘Ae Meri Zohra Jabeen’ from the film ‘Waqt’who passed away recently. Achala Sachdev passed away at a Pune hospital on April 30, 2012. was 91 year old. 2. Where in India, largest solar power plant with an installed capacity of 40 MW started producing electricity? Pokhran in Jaisalmer district of Rajasthan 3. Which department in India plans to set up 1000 ATM’s across 6 states? Postal department 4. Who created history by becoming the first Indian female wrestler to have qualified for the London Olympics 2012? Geeta 5. Who has been appointed next Chairman of Atomic Energy Commission? Ratan Kumar Sinha 6. In which part of India, Indo Tibetan Border Police headquarters is to be opened? Bihar 7. Which state’s High Court has insisted that banks cannot freeze accounts nor stop services of issuing cheque book or ATM facility where the account holder has not supplied KYC (Know Your Customers) documents? Gujarat 8. Name the former Haryana chief secretary who took oath as state information commissioner ? Urvashi Gulati 9. Which State becames the 3rd state in India to launch State Portal (SP) and State Service Delivery Gateway (SSDG) project? Nagaland 10. Who won the World Series Hockey 2012 title? Sher-E-Punjab 11. In which state Urdu medium Government schools would be setup as announced by Chief Minister Akhilesh Yadav? Uttar Pradesh 12. Where is the World’s first Virtual shopping store opened? Korea 13. Who is named as HSBC’s India CEO? Stuart Milne 14. Which Indian state bans guthka from 1st of April 2012 to curb the increasing number of mouth cancer cases? Madhya Pradesh 15. Which Indian state chief minister has urged union Finance Minister to bail it out of its huge burden? West Bengal 16. Name the Jammu and Kashmir High Court Chief Justice who was sworn in as Supreme court judge ? F.M.Ibrahim Kalifulla 17. Name the Gujarati techie whose name has emerged for India’s President as the nation has to elect 16th President in July 2012 ? Sam Pitroda 18. At what value Air India’s restructuring plan was approved by a group of 19 banks led by SBI? 18,000 crore 19. Which services Airtel is planning to launch in Kolkata in April 2012? 4G services 20. Name the nuclear submarine which joined Indian Navy. INS Chakra 21. Which education company bagged Rs.209 crore order from Assam Government? Educomp 22. Which 2 north Indian states will have pipeline connected oil terminals for supply of diesel, petrol and kerosene? Jammu and Himachal Pradesh 23. Which Asian country has issued tender to buy minimum of 30,000 tonnes of basmati rice from India and Pakistan? Iraq 24. NSE founder who passes away Recently? RH PatilDr R H Patil, chairman, Clearing Corporation of India (CCIL) and founder-managing director of the National Stock Exchange (NSE) passed away in Mumbai 25. Who donated 1 million Singapore Dollars to Indian Heritage Centre? 26. Name the world’s highest and longest bridge that was officially opened to motorists in China.Anzhaite Long-span Suspension Bridge in Jishou 27. Which bank has opened 16 Ultra Small branches in 16 locations in villages to cater to financial needs of the rural populace? IDBI Bank 28. Who has been conferred with Sir Jehangir Ghandy Medal? Narayana Murthy 29. In which city in Kerala will Infosys setup its 2nd campus? Thiruvananthapuram 30. Which bank will set up 1-person micro branches to serve remote areas of the northeastern region financially? Indian Overseas Bank 31. Which act does not allow to prosecute (no wife can claim compensation or maintenance) husband’s girlfriend? Protection of Women from Domestic Vilonce Act, 2005 32. Name the 3 Indian hockey players after whom tube stations in London has been renamed.Dhyan Chand, Roop Singh and Leslie Claudius 33. Who has been signed up as brand ambassador of Toyota Kirloskar Motors? Virat Kohli 34. When was 63rd Rajasthan Day celebrated? 30th March 2012 35. Where in Rajasthan Spices Board’s first Spices Park is setup? Jodhpur 36. Which Asian country recently became member of Consultative Group to Assist the Poor (CGAP)? India 37. Who won her 6th world sprint title at the world track cycling championships 2012? Victoria Pendleton 38. Name the Malawi President who died after heart attack. Bingu wa Mutharika 39. Who was sworn in as Malawi’s President following the death of Bingu wa Mutharika? Joyce Banda 40. Which Asian country made Hindu marriage registration a must by bringing the law in lines with India? Bangladesh 41. Israel has decided to setup its consulate office in which part of India? Bangalore 42. Which Indian city is likely to get water taxis and airconditioned trams? Kolkata 43. Name the young doctor who is set to become the first foreign national of Indian origin to walk across Antarctica carrying the Indian Flag. Dr.Alexander Kumar 44. Name the youngest Indian boxer who qualified for London Olympics 2012. Shiva Thapa 45. What percentage hike was allocated for Karnataka by Planning Commission for current fiscal 2012-13 in the 12th Plan? 10% 46. Who will be the sports ambassador for the state of Haryana? Leander Paes 47. Which category of women will be allowed to participate in the Miss Universe beauty pageant from next year? Transgender women 48. Which rank does Bangalore hold as preferred entrepreneurial locations worldwide? 9th rank 49. Which TV has become the first Indian television channel to enter the Chinese market? Zee TV 50. Which act was amended to include compulsory registration of marriages in India? Registration of Births and Deaths Act, 1969 51. Where in Himachal Pradesh Indian Institute of Information Technology (IIIT) would be opened? Una district 52. What denominations of currency note will RBI introduce bearing rupee symbol? Rs.20 and Rs.50 currency notes 53. Name the index on which NSE is set to launch derivative trade. FTSE 100 index 54. What percentage and value of US based Max New York Life Insurance was sold to Mitsui Sumitomo Insurance (MSI)? 26% and Rs.2,731 crore 55. Which country replaced China as Iran’s top oil client? India 56. India has recently decided to allow Foreign Direct Investment from which Asian country? Pakistan 57. What value of equity infusion did Air India get spread over a period of 9 years? Rs. 30,000 crore 58. Where in India Integrated Check Post was inaugurated on the occasion of Baisakhi to enhance trade between India-Pakistan? Attari 59. Which education council in India would have its own job portal for the students of technical and professional institutions? All India Council for Technical Education (AICTE) 60. Which 2 countries have agreed to resume dialogue on disarmament and non-proliferation after a gap of 9 years? India and China 61. Which repayment transaction is now allowed by RBI electronically i.e. NEFT? Repayment of Loan EMI’s 62. How many co-sponsors are against the Bill of outsourcing call centers in USA? 106 co-sponsors 63. How much basis point did RBI cut key rates after 3 years? 50 basis points 64. Name the Master Card’s President and CEO who has been elected next chairman of the US-India Business Council? Ajay Banga 65. Which Indian FMCG company crossed $1 billion mark in 2011-12? Dabur India 66. Which 14 year old ICC cricket tournament would end after 2013? ICC Champions Trophy 67. Which state has included services of the petroleum industry, including that provided by tanker operations as essential services under the state’s Essential Services Maintenance Act 1971, (Act 20 of 1971)? Andhra Pradesh 68. Which Indian state’s 100th anniversary was celebrated recently? Bihar 69. Who replaces Simon Leung chief executive of the greater China region of Microsoft? Ralph Haupter 70. Bill to include children with disabilities under which Act was recently passed by Rajya Sabha recently? Right to Education Act 71. Name the spy satellite launched by ISRO recently.RISAT-1 72. Which cricket player and lady actor were recommended for Rajya Sabha membership in India? Sachin Tendulkar and Rekha 73. Name the Indian origin entrepreneur and educator who is named as the Goodwill Ambassador for Education Partnerships for UN Educational, Scientific and Cultural Organisation (UNESCO). Sunny-Varkey 74. Who has been appointed as Governor of Goa, Rajasthan and Uttarkhand? B.V.Wanchoo, Margaret Alva and Aziz Qureshi 75. Who were given fresh terms as Governor of Andhra Pradesh and Maharashtra? E.S.L.Narasimhan and K.Sankaranarayanan 76. Which Indian state will be showcased at the Cannes Film Festival? Gujarat 77. Which 2 Asian countries will hold 1st ever economic dialogue on 30th April 2012? India and Japan 78. Name the first of the four Dreamliner aircraft that Air India got from Nikki Haley’s state.Boeing 78779. Who won the Asian Snooker Championship 2012? Aditya Mehta 80. Who retained the crown as Britain’s richest man? Lakshmi Mittal 81. Where was the India Investrade 2012 organised by Indian Chamber of Commerce, Kolkata held? Colombo, Sri Lanka 82. Name the mango variety named after the youngest Chief Minister of Uttar Pradesh. ‘Akhilesh aam’, named after Akhilesh Yadav 83. Which Indian state is planning to make 100 the common number to dial in any emergency? Tamil Nadu 84. Who became the 1st player to cross 2000 runs in cricket tournament IPL? Suresh Raina 85. Which country is keen to boost trade ties with Kerala? Singapore 86. Who became the 1st player to win Barcelona title for 7 times? Rafael Nadal 87. Name the former BCCI president who passed away recently after whom the challenger trophy was named. N.K.P.Salve 88.Who won the Champions Trophy women’s hockey tournament for the 5th time? Argentina 89.Who has been appointed as Chief executive operations for China’s Walmart? Greg Foran 90.When is Windows 8 set to be released by Microsoft? February 29th 2012 UPSC History questions UPSC written test examination questions with answers UPSC General Awareness questions with answers UPSC different written test examination questions papers,UPSC Previous year solved Question paper with Answer, UPSC Solved Question paper for Assistant Commandants Exam 2012,UPSC model Question paper with Answer, UPSC Latest Assistant Commandants Exam details, UPSC Latest recruitment of Assistant Commandants 2012 latest Selection pattern, Written Exam details, UPSC Latest Selection procedure for Central Armed Police Force Exam 2012, UPSC General Ability and intelligence Questions with Answers, New UPSC Written Exam Syllabus for Assistant commandant Exam 2012, UPSC General studies Questions and comprehension Question pattern for recruitment of Assistant Commandant Exam 2012 UPSC 100 History based general awareness and current affairs 1. The Battle of Plassey was fought in A.1757 B.1782 C.1748 D.1764 Answer: Option A Explanation: The Battle of Plassey, 23 June 1757, was a decisive British East India Company victory over the Nawab of Bengal and his French allies, establishing Company rule in South Asia which expanded over much of the Indies for the next 190 years. The battle took place at Palashi, Bengal, on the river banks of the Bhagirathi River, about 150 km north of Calcutta, near Murshidabad, then capital of undivided Bengal. The belligerents were Siraj-ud-daulah, the last independent Nawab of Bengal, and the British East India Company. More info: http://en.wikipedia.org/wiki/Battle_of_Plassey 2. The territory of Porus who offered strong resistance to Alexander was situated between the rivers of A.Sutlej and Beas B.Jhelum and Chenab C.Ravi and Chenab D.Ganga and Yamuna Answer: Option B 3. Under Akbar, the Mir Bakshi was required to look after A.military affairs B.the state treasury C.the royal household D.the land revenue system Answer: Option A Explanation: The head of the military was called the Mir Bakshi, appointed from among the leading nobles of the court. The Mir Bakshi was in charge of intelligence gathering, and also made recommendations to the emperor for military appointments and promotions. 4. Tripitakas are sacred books of A.Buddhists B.Hindus C.Jains D.None of the above Answer: Option A 5. The trident-shaped symbol of Buddhism does not represent A.Nirvana B.Sangha C.Buddha D.Dhamma Answer: Option E 6. The theory of economic drain of India during British imperialism was propounded by A.Jawaharlal Nehru B.Dadabhai Naoroji C.R.C. Dutt D.M.K. Gandhi Answer: Option D 7. The treaty of Srirangapatna was signed between Tipu Sultan and A.Robert Clive B.Cornwallis C.Dalhousie D.Warren Hastings Answer: Option A 8. The system of competitive examination for civil service was accepted in principle in the year A.1833 B.1853 C.1858 D.1882 Answer: Option E 9. Through which one of the following, the king exercised his control over villages in the Vijayanagar Empire? A.Dannayaka B.Sumanta C.Nayaka D.Mahanayakacharya Answer: Option B 10. The Vijayanagara ruler, Kirshnadev Raya's work Amuktamalyada, was in A.Telugu B.Sanskrit C.Tamil D.Kannada Answer: Option A 11. Under an agreement with which of the following countries did Subhas Chandra Bose organize the Indian soldiers, taken as prisoners by the Axis Powers, into the Azad Hind Fauj? A.China B.Germany C.Italy D.Japan Answer: Option E 13. The use of Kharoshti in ancient Indian architecture is the result of India's contact with A.Central Asia B.Iran C.Greece D.China Answer: Option B 14. Vaikhanasa the five-fold conception of Vishnu consists ofbrahman purusha prakriti satya achyuta aniruddha A.I, II, III, IV and V B.II, III, IV, V and VI C.I, II, IV, V and VI D.I, III, IV, V and VI 15. The troops raised by the emperor but not paid directly the state and place under the charge of mansabadars were know as A.Walashahi B.Barawardi C.JCumaki D.Dakhili Answer: Option C 16. The treaty of Mangalore was signed between A.the English East India Company and Haidar Ali B.the English East India Company and Tipu Sultan C.Haidar Ali and the Zamorin of Calicut D.the French East India Company and Tipu Sultan 17. To conquer and annex Peshawar and Punjab, Mahmud of Ghazni defeated A.Ghurids B.Arabs C.Karkotakas D.Hindushahis Answer: Option C 18. To which professions earlier leaders who struggled for freedom of India mainly belonged? A.Lawyers B.Teachers C.Journalists D.All of the above Answer: Option E 19. The victories of Karikala are well portrayed in A.Palamoli B.Aruvanad C.Pattinappalai D.Padirrupattu Answer: Option D 20. Todar Mal was associated with A.music B.literature C.land revenue reforms D.law Answer & Explanation Answer: Option A 21. The title of 'Viceroy' was added to the office of the Governor-General of India for the first time in A.1848 AD B.1856 AD C.1858 AD D.1862 AD Answer: Option C 22. To which of the following dynasties did Kind Bhoja, a great patron of literature and art, belong? A.Karkota B.Utpala C.Paramara D.Gurjara Pratihara Answer: Option D 23. Vikramaditya, a king of Ujjain, started the Vikrama samvat in 58 BC in commemoration of his victory over A.Indo-Greeks B.Sakas C.Parthinas D.Kushanas Answer: Option B 24. Two of the great Mughals wrote their own memories. There were A.Babar and Humayun B.Humayun and Jahangir C.Babar and Jahangir D.Jahangir and Shahjahan 25. To which king belongs the Lion capitol at Sarnath? A.Chandragupta B.Ashoka C.Kanishka D.Harsha Answer: Option B 26. The use of spinning wheel (Charkha) became common during the A.9th Century AD B.10th Century AD C.12th Century AD D.14th Century AD Answer: Option D 27. The language of discourses of Gautama Buddha was A.Bhojpuri B.Magadhi C.Pali D.Sanskrit Answer: Option B 28. There were widespread risings against the British in the 1820s. Which one of the following did not revolt in the 1820s? A.Santhals B.Ahoms C.Pagal Panthis D.Ramosi Answer: Option E 29. Velu Thampi led a revolt against the British in state o fA.Travancore B.Baroda C.Hyderabad D.Mysore Answer: Option C 30. Under the Mountbatten Plan of 1947 the people of ___ were given the right to decide through a plebiscite whether they wished to join Pakistan or India. A.Assam B.Punjab C.Bengal D.N.W.F.P and the Sylhet district of Assam Answer: Option E 31Three major powers that emerged in southern India in the 7th century AD wereCheras Cholas Chalukyas Pallavas Pandyas A.I, II, V B.II, III, IV C.III, IV, V D.I, II, IV Answer: Option A 32. The term 'Yavanapriya' mentioned in ancient Sanskrit texts denoted A.ivory B.pepper C.a fine variety of Indian muslin D.damsels sent to the Greek court for dance performance Answer: Option E 33. The Timariots Governors and the Revenue Contractors, on their part reason in this manner: "Why should the neglected state of this land create uneasiness in our minds and why should we expend our money and time to render it fruitful? We may be deprived of it in a single moment, and our exertions would benefit neither ourselves nor our children." This statement was made by A.Monserrate B.Tavernier C.Manrique D.Bernier Answer: Option A 34. The ultimate ownership of land during the post-Gupta period lay with A.the cultivator B.the village community C.the king D.the joint family Answer: Option A 35. To which of the republic of Buddha belong? A.Licchavis B.Sakyas C.Mallas D.None of the above Answer: Option E 36. There was a sharp class division at Harappa and Mohen-jodaro. This is clear from the A. Indus seals excavated B. religious beliefs of the Harappans C. tools and implements used by the Harappans D. different types of dwellings excavated Answer & Explanation Answer: Option A Explanation: No answer description available for this question. Let us discuss. Workspace 37. The title given by the British Government to Mahatma Gandhi which he surrendered during the non-cooperation movement was A.Hind Keasri B.Kaiser-e-Hind C.Rai Bahadur D.Rt. Honorable Answer: Option E 38. Tipu sultan was the ruler of A.Hyderabad B.Madurai C.Mysore D.Vijayanagar Answer: Option B 39. The term yavanika meant A.foreign goods B.dancer C.curtain D.theatre Answer: Option C 40. The term Khalisa in Mughal administration signified the A.entire Imperial establishment B.land owned b the emperor himself C.religious land grants D.land from where revenue was collected for the Imperial Treasury Answer: Option E 41Visakhadatta sketches the event after the death of Samudragupta in his work A.Mudrarakasam B.Devi Chand Guptam C.Mrichekakatika D.Malavikagnimitra 42. The system of Dual Government during the latter half of the 18th century AD is associated with the name of A.Clive B.Comwallis C.Waren Hastings D.William Bentinck Answer: Option C 43. 'The Vedas contain all the truth was interpreted by A.Swami Vivekananda B.Swami Dayananda C.Raja Rammohan Roy D.None of the above 44. The term samanta, meaning a feudatory from the sixth century AD, originally meant a A.slave B.cultivator C.neighbor D.foreigner 45. To evolve a peaceful settlement of the conflict between India and China, which of the following non-aligned Afro-Asian nations participated in a conference held in December 1962? A.Burma (now Myanmar), Combodia, Indonesia and UAR B.Burma, Sri Lanka, Combodia and Indonesia C.Burma, Indonesia, Ghana and Sri Lanka D.All of the above 46 The text of the document called Mahzar, by which Akbar assumed the role of supreme arbiter in the matters of religion is found in A.Nizamuddin's Tabaqat-I-Akbari B.Arif Quandahari's Tarikh-I-Alfi C.Abul Fazl's Akbarnama D.Badauni's Muntakahab-ut-Tawarikh Answer: Option C 47The Vedic deity Indra was the Goddess of A.wind B.eternity C.rain and thunder D.fire Answer: Option C 47. Tulsidas, the author of Ramcharitmanas, was a contemporary of which of the following rulers? A.Akbar B.Humayun C.Shahjahan D.Sher Shah Suri Answer: Option C 48. To meet the educational needs of the people, the Madarasa-I Nasiri was built in the region of A.Qutub-ud-din Aibak B.Iltutmish C.Ruknuddin Firoz Shah D.Jalal-id-din Khilji Answer: Option C 49. The weekly Commonweal was founded by A.Annie Besant B.Bipan Chandra Pal C.Bal Gangadhar Tilak D.Sarojini Naidu Answer: Option E 50. Ustad Mansur was a famous painter in the region of A.Shajahan B.Akbar C.Humayun D.Jahangir Answer: Option B 51. The Vedic deity Indra was the Goddess of A. wind B. eternity C. rain and thunder Dfire Answer: Option C 52. Tolkappiyam is associated with the A.first Sangam period B.second Sangam period C.third Sangam period D.post-third Sangam period 53. Pulakesin II was the most famous ruler ofA. Chalukyas B. Cholas C. Pallavas D. Satavahanas Answer & Explanation Answer: Option C Explanation: No answer description available for this question. Let us discuss. Workspace 54. The term Brahmadeya occurs for the first time in A.early Vedic texts B.early Buddhist texts C.pre-Gupta inscriptions D.post-Gupta inscriptions Answer: Option E 55. Under whose leadership was the all India Muslim League set up? A.Mohammed Ali Jinnah B.Sayyid Ahmed Khan C.Aga Khan D.All of the above E.None of the above Answer: Option C 56. Though Ashoka had many sons, the inscriptions mentioned only one who is not mentioned in any other source. He is A.Kunala B.Tivara C.Mahendra D.Jalauka Answer: Option D 57. We can know about early vedic period from A.archaeological excavations B.the Rig Veda C.Jatak Katha D.contemporary culture Answer: Option A 58. The Upanishads are A.a source of Hindu philosophy B.books of ancient Hindu laws C.books on social behavior of man D.prayers to God Answer: Option A 59. Universities in the Presidency towns in India were established in A.1857 B.1858 C.1900 D.1909 Answer: Option D 60. The Vijayanagara king who employed skilled archers of the Turkish clan and raised the fighting capacity of his bowmen was A.Bukka I B.Devaraya I C.Krishnadevaraya D.Ramaraya Answer: Option D 61. Under the Guptas in eastern India, there was probably an intermediate level of administration between vishayas (districts) and villages. Identify it. A.Bhukit B.Pradesa C.Vifhi D.Ahara Answer: Option A 62. The two principles monuments of Alaud-din Khilji's reign - the Jama at Kana Masjid and Alai Darwaza - were constructed at A.Agra B.Delhi C.Dhar D.Gulbarga Answer: Option C 63. The term Nirgrantha is associated with A.Ajivikas B.Charvakas C.Jainas D.Pasupatas Answer: Option E 64. The Kalinga was fought in A.321 BC B.301 BC C.261 BC D.241 BC Answer: Option E 65. Under the Government of India, Provincial Legislatures consisted of two chambers, except in the case of A.Assam B.Bihar C.Madras D.Punjab Answer: Option E 66. The Venetian traveler who travelled with his wife and reached Vijayanagar around 1420 was A.Athanasius Nikitin B.Nicolo de Conti C.Ibn Batuta D.Ferishta Answer: Option E 67. The year 788 AD was a good one for Hinduism. Why? A.Shankracharya was born that year. B.Harsha Vardhana, the last Buddhist king, died. C.Samudragupta converted to Hinduism. D.All Muslim invaders were defeated. Answer: Option E 68. The Uprising of 1857 was described as the first Indian war of Independence by A.S.N. Sen B.R.C. Mazumdar C.B.G. Tilak D.V.D. Savakar Answer: Option B 69. The twenty-third Jaina teacher, Parsva, the immediate predecessor of Mahavira enjoined on his disciples four great vows. To these Mahavira addes which of the followings as the fifth vow? A.Abstention from stealing B.Non-injury C.Brahmacharya or continence D.Non-attachment Answer: Option B 70. The Turko-Afghan rule in India lasted for about A.two centuries B.three centuries C.four centuries D.a little over one century Answer: Option E 71Which is the oldest parliamentary democracy in the world? Great Britain 72 What is the holy book of Parsis called? The Zend Avesta 73 Which city was founded by Alexander the Great in Egypt? Alexandria 74 Who was known as Maid Of Orleans? Joan of Arc 75 Which American President said: "It's best not to swap horses while crossing the river"? Abraham Lincoln 76 What is the Japanese parliament called? Diet 77 Who was the German-Jewish girl who hid in an Amsterdam attic during WW II and wrote a diary which later became famous? Anna Frank 78 Of which British King was it said, "He never said a foolish thing, nor ever did a wise one"? King Charles II 79 Which Roman emperor build Byzantium? Constantine 80 Which is the largest pyramid? The Great Pyramid was built for King Cheops (Khufu), the second King of the IVth Dynasty. 81 After which Florentine explorer was America named? Amerigo Vespucci 82 What is Bahai'ism? A religion founded by Baha'ullah in Iran during 19th century. Baha'ists believe in the unity of all religions, in universal education, in world peace and the equality of men and women. 83In which century was Lord Buddha born? 6th Century BC (around 560 BC). 84 What was Abraham Lincoln's profession prior to entering Congress? Lawyer 85 Which famous Englishman was killed in the Battle of Trafalgar? Admiral Lord Nelson 86 Who are the Cossacks? People of Southern Russia famous as horsemen and cavalrymen. 87 Which are three magic words of the French revolution? Egalite', Liberte', Fraternite' (Equality, Liberty, Fraternity) 88. “Stored Program Concept” postulated by: Ans. John von neumann 89. Founder of the social networking website ‘Facebook’: Ans. Mark Elliot Zuckerberg 90 . Name associated with Open Source Program: Ans. Richard Stallman 91 The main routes that data travels over the internet is called: Ans. Internet backbone 92 An IT term refers to allowing someone to open web pages and see web sites on the internet. Ans. Web Surfing 93 An internet protocol that allows quick file tarnsmission to remote computers is know as: Ans. FTP 94 Protocol used to temporarily store new messages in your mailbox on an e-mail server is known as: Ans. POP 95 .Company which maintains internet computers and telecommunications equipment in order to provide internet access to businesses, organizations and individuals is called: Ans. Internet Service Provider 96 The device that protects all computers in the network from many attacks is called: Ans. Firewall 97 . Free telephone style conversations to travel over the internet to virtually anywhere in the world is known as: Ans. Internet Telephony 98 Who was the last pharoah of Egypt? Cleopatra VII 99 Who was the leader of Russia during World War Two? Josef Stalin 100 Who is credited with being the first European to discover North America? Christopher Columbus UPSC Question-Paper UPSC Civil Service Prelims Solved Question Paper General Studies: 1. Priority Sector Lending by banks in India constitutes the lending to (a) agriculture (b) micro and small enterprises (c) weaker sections (d) All of the above 2. Which one among the following industries is the maximum consumer of water in India? (a) Engineering (b) Paper and pulp (c) Textiles (d) Thermal power 3. To obtain full benefits of demographic dividend, what should India do? (a) Promoting skill development (b) Introducing more social security schemes (c) Reducing infant mortality rate (d) Privatization of higher education 4. In the context of cultural history of India, a pose in dance and dramatics called ?Tribhanga? has been a favourite of Indian artists from ancient times till today. Which one of the following statements best describes this pose? (a) One leg is bent and the body is slightly but oppositely curved at waist and neck (b) Facial expressions, hand gestures and make-up are combined to symbolize certain epic or historic characters (c) Movements of body, face and hands are used to express oneself or to tell a story (d) A little smile, slightly curved waist and certain hand gestures are emphasized to express the feelings of love or eroticism. 5. Annie Besant was 1. Responsible for starting the Home Rule Movement 2. The founder of the Theosophical Society 3. Once the President of the Indian National Congress Select the correct statement / statements using the codes given below (a) 1 only (b) 2 and 3 only (c) 1 and 3 only (d) 1, 2 and 3 6. The Ilbert Bill controversy was related to the (a) imposition of certain restrictions to carry arms by the Indians (b) imposition of restrictions on newspapers and magazines published in Indian languages (c) removal of disqualifications imposed on the Indian magistrates with regard to the trial of the Europeans (d) removal of a duty on imported cotton cloth. 7. A rise in general level of prices may be caused by 1. an increase in the money supply 2. a decrease in the aggregate level of output 3. an increase in the effective demand Select the correct answer using the codes given below. (a) 1only (b) 1and 2 only (c) 2 and 3 only (d) 1, 2 and 3 8. Which one of the following groups of items is included in India?s foreign ? exchange reserves? (a) Foreign-currency assets, Special Drawing Rights (SDRs) and loans from foreign countries (b) Foreign ? currency assets, gold holdings of the RBI and SDRs (c) Foreign ? currency assets, loans from the World Bank and SDRs (d) Foreign ? currency assets, gold holdings of the RBI and loans from the World Bank 9. Which one of the following is likely to be the most inflationary in its effect? (a) Repayment of public debt (b) Borrowing from the public to finance a budget deficit (c) Borrowing from banks to finance a budget deficit (d) Creating new money to finance a budget deficit 10. Supply of money remaining the same when there is an increase in demand for money, there will be (a) a fall in the level of prices (b) an increase in the rate of interest (c) a decrease in the rate of interest (d) an increase in the level of income and employment 11. Fruits stored in a cold chamber exhibit longer storage life because (a) Exposure to sunlight is prevented (b) Concentration of carbon dioxide in the environment is increased (c) Rate of respiration is decreased (d) There is an increase in humidity 12. Consider the following fauna of India: 1. Gharial 2. Leatherback turtle 3. Swamp deer Which of the above is/are endangered? (a) 1 and 2 only (b) 3 only (c) 1, 2 and 3 (d) None 13. Ball bearings are used in bicycles, cars, etc., because (a) the actual area of contact between the wheel and axle is increased (b) the effective area of contact between the wheel and axle is increased (c) the effective area of contact between the wheel and axle is reduced (d) None of the above statements is correct 14. Consider the following phenomena: 1. Size of the sun at dusk 2. Colour of the sun at dawn 3. Moon being visible at dawn 4. Twinkle of stars in the sky 5. Polestar being visible in the sky Which of the above are optical illusions? (a) 1, 2 and 3 (b) 3, 4 and 5 (c) 1, 2 and 4 (d) 2, 3 and 5 15. Rainbow is produced when sunlight falls on drops of rain. Which of the following physical phenomena are responsible for this? 1. Dispersion 2. Refraction 3. Internal reflection Select the correct answer using the codes given below (a) 1 and 2 only (b) 2 and 3 only (c) 1 and 3 only (d) 1, 2 and 3 16. Many transplanted seedling do not grow because (a) the new soil does not contain favourable minerals (b) most of the root hairs grip the new soil too hard (c) most of the root hair are lost during transplantation (d) leaves get damaged during transplantation 17. Economic growth in country X will necessarily have to occur if (a) there is technical progress in the world economy (b) there is population growth in X (c) there is capital formation in X (d) the volume of trade grows in the world economy 18. Which of the following statements is/are correct 1. Viruses lack enzymes necessary for the generation of energy 2. Viruses can be cultured in any synthetic medium 3. Viruses are transmitted from one organism to another by biological vectors only. Select the correct answer using the code given below codes (a) 1only (b) 2 and 3 only (c) 1 and 3 only (d) 1, 2 and 3 19. Which of the following leaf modification occurs/occur in desert areas to inhibit water loss? 1. Hard and waxy leaves 2. Tiny leaves or no leaves 3. Thorns instead of leaves Select the correct answer using the codes given below: (a) 1 and 2 only (b) 2 only (c) 1 and 3 only (d) 1, 2 and 3 20. The known forces of nature can be divided into four classes, viz., gravity, electromagnetism, weak nuclear force and strong nuclear force. With reference to them, which one of the following statements is not correct? (a) Gravity is the strongest of the four (b) Electromagnetism acts only on particles with an eclectic charge (c) Weak nuclear force causes radioactivity (d) Strong nuclear force holds protons and neutrons inside the nucleus of an atom 21. The efforts to detect the existence of Higgs boson particle have become frequent news in the recent past. What is/are the importance /importances of discovering this particle? 1. It will enable us to understand as to why elementary particles have mass. 2. It will enable us in the near future to develop the technology of transferring matter from one point to another without traversing the physical space between them. 3. It will enable us to create better fuels for nuclear fission. Select the correct answer using the code given below (a) 1 only (b) 2 and 3 only (c) 1 and 3 only (d) 1, 2 and 3 22. Mycorrhizal biotechnology has been used in rehabilitating degraded sites because mycorrhiza enables the plants to 1. Resist drought and increase absorptive area 2. Tolerate extremes of pH 3. Resist disease infestation Select the correct answer using the code given below (a) 1 only (b) 2 and 3 only (c) 1 and 3 only (d) 1, 2 and 3 23. Who among the following constitute the National Development Council? 1. The Prime Minister 2. The Chairman, Finance Commission 3. Ministers of the Union Cabinet 4. Chief Ministers of the States Select the correct answer using the code given below (a) 1, 2 and 3 only (b) 1, 3 and 4 only (c) 2 and 4 only (d) 1, 2, 3 and 4 24. The national income of a country for a given period is equal to the (a) Total value of goods and services produced by the nationals. (b) Sum of total consumption and investment expenditure. (c) Sum of personal income of all individuals. (d) Money value of final goods and services produced. 25. Which of the following grants/grant direct credit assistance to rural households? 1. Regional Rural Banks 2. National Bank for Agriculture and Rural Development. 3. Land Development Banks. Select the correct answer using the code given below (a) 1 and 2 only (b) 2 only (c) 1 and 3 only (d) 1, 2 and 3 26. Consider the following statements: The Parliamentary Committee on Public Accounts 1. consists of not more than 25 Members of the Lok Sabha. 2. scrutinizes appropriation and finance accounts of the Government. 3. examines the report of the Comptroller and Auditor General of India. Which of the statements given above is/are correct? (a) 1 only (b) 2 and 3 only (c) 3 only (d) 1, 2 and 3 27. Consider the following Bhakti Saints: 1. Dadu Dayal 2. Guru Nanak 3. Tyagaraja Who among the above was/were preaching when the Lodi dynasty fell and Babur took over? (a) 1 and 3 (b) 2 only (c) 2 and 3 (d) 1 and 2 28. With reference to the food chains in ecosystems, which of the following kinds of organism is/are known as decomposer organism/organisms? 1. Virus 2. Fungi 3. Bacteria Select the correct answer using the code given below (a) 1 only (b) 2 and 3 only (c) 1 and 3 only (d) 1, 2 and 3 29. The most important fishing grounds of the world are found in the regions where (a) warm and cold atmospheric currents meet (b) rivers drain out large amounts of freshwater into the sea (c) warm and cold oceanic currents meet (d) continental shelf is undulating 30. Which of the following is/are unique characteristic/ characteristics of equatorial forests? 1. Presence of tall, closely set trees with crowns forming a continuous canopy. 2. Coexistence of a large number of species. 3. Presence of numerous varieties of epiphytes. Select the correct answer using the code given below (a) 1 only (b) 2 and 3 only (c) 1 and 3 only (d) 1, 2 and 3 31. Which of the following constitute Capital Account 1. Foreign Loans 2. Foreign Direct Investment 3. Private Remittances 4. Portfolio Investment Select the correct answer using the codes given below (a) 1, 2 and 3 (b) 1, 2 and 4 (c) 2, 3 and 4 (d) 1, 3 and 4 32. Consider the following historical places: 1. Ajanta Caves 2. Lepakshi Temple 3. Sanchi Stupa Which of the above places is/are also known for mural paintings? (a) 1 only (b) 1 and 2 only (c) 1, 2 and 3 (d) None 33. With reference to the history of philosophical thought in India, consider the following statements regarding Sankhya school: 1. Sankhya does not accept the theory of rebirth or transmigration of soul. 2. Sankhya holds that it is the self-knowledge that leads to liberation and not any exterior influence or agent. Which of the statements given above is/are correct? (a) 1 only (b) 2 only (c) Both 1 and 2 (d) Neither 1 nor 2 34. In the context of India, which of the following principles is/are implied institutionally in the parliamentary government? 1. Members of the Cabinet are Members of the Parliament. 2. Ministers hold the office till they enjoy confidence in the Parliament. 3. Cabinet is headed by the Head of the State. Select the correct answer using the codes given below (a) 1 and 2 only (b) 3 only (c) 2 and 3 only (d) 1, 2 and 3 35. The annual range of temperature in the interior of the continents is high as compared to coastal areas. What is/are the reason/reasons? 1. Thermal difference between land and water. 2. Variation in altitude between continents and oceans. 3. Presence of strong winds in the interior. 4. Heavy rains in the interior as compared to coasts. Select the correct answer using the codes given below: (a) 1 only (b) 1 and 2 only (c) 2 and 3 only (d) 1, 2, 3 and 4 36. Which of the following is/are the characteristic/ characteristics of Indian coal? 1. High ash content 2. Low sulphur content 3. Low ash fusion temperature Select the correct answer using the codes given below (a) 1 and 2 only (b) 2 only (c) 1 and 3 only (d) 1, 2 and 3 37. Which of the following statements regarding laterite soils of India are correct? 1. They are generally red in colour. 2. They are rich in nitrogen and potash. 3. They are well-developed in Rajasthan and UP. 4. Tapioca and cashew nuts grow well on these soils. Select the correct answer using the codes given below (a) 1, 2 and 3 (b) 2, 3 and 4 (c) 1 and 4 (d) 2 and 3 only 38. Consider the following statements: 1. Natural gas occurs in the Gondwana beds 2. Mica occurs in abundance in Kodarma 3. Charwars are famous for petroleum Which of the statements given above is/are correct? (a) 1 and 2 (b) 2 only (c) 2 and 3 (d) None 39. Consider the following crops: 1. Cotton 2. Groundnut 3. Rice 4. wheat Which of these are Kharif crops? (a) 1 and 4 (b) 2 and 3 only (c) 1 2, and 3 (d) 2, 3 and 4 40. ?Climate is extreme, rainfall is scanty and the people used to be nomadic herders?. The above statement best describes which of the following regions? (a) African Savannah (b) Central Asian Steppe (c) North American Prairie (d) Siberian Tundra 41. Consider the following statements: 1. Inflation benefits the debtors. 2. Inflation benefits the bond-holders. Which of the statements given above is/are correct? (a) 1 only (b) 2 only (c) Both 1 and 2 (d) Neither 1 nor 2 42. Disguised unemployment generally means (a) large number of people remain unemployed (b) alternative employment is not available (c) marginal productivity of labour is zero (d) productivity of workers is low 43. Consider the following statements: 1. The Council of Ministers in the Centre shall be collectively responsible to the Parliament. 2. The Union Ministers shall hold the office during the pleasure of the President of India. 3. The Prime Minister shall communicate to the President about the proposals for legislation. Which of the statements given above is/are correct? (a) 1 only (b) 2 and 3 only (c) 1 and 3 only (d) 1, 2 and 3 44. Consider the following statements: 1. National Development Council is an organ of the Planning Commission. 2. The Economic and Social Planning is kept in the Concurrent List in the Constitution of India. 3. The Constitution of India prescribes that Panchayats should be assigned the task of preparation of plans for economic development and social justice. Which of the statements given above is/are correct? (a) 1 only (b) 2 and 3 only (c) 1 and 3 only (d) 1, 2 and 3 45. Consider the following statements: 1. The Chairman and the Deputy Chairman of the Rajya Sabha are not the members of that House. 2. While the nominated members of the two Houses of the Parliament have no voting right in the presidential election, they have the right to vote in the election of the Vice President. Which of the statements given above is/are correct? (a) 1 only (b) 2 only (c) Both 1 and 2 (d) Neither 1 nor 2 46. With reference to National Legal Services Authority, consider the following statements: 1. Its objective is to provide free and competent legal services to the weaker sections of the society on the basis of equal opportunity. 2. It issues guidelines for the State Legal Services Authorities to implement the legal programmes and schemes throughout the country. Which of the statements given above is/are correct? (a) 1 only (b) 2 only (c) Both 1 and 2 (d) Neither 1 nor 2 47. During a thunderstorm, the thunder in the skies is produced by the 1. meeting of cumulonimbus clouds in the sky. 2. lightning that separates the nimbus clouds. 3. violent upward movement of air and water particles. Select the correct answer using the code given below. (a) 1 only (b) 2 and 3 only (c) 1 and 3 (d) None of the above produces the thunder 48. Consider the following pairs: Tribe State 1. Limboo (Limbu): Sikkim 2. Karbi: Himachal Pradesh 3. Dongaria Kondh: Odisha 4. Bonda: Tamil Nadu Which of the above pairs are correctly matched? (a) 1 and 3 only (b) 2 and 4 only (c) 1, 3 and 4 only (d) 1, 2, 3 and 4 49. Consider the following liquid assets: 1. Demand deposits with the banks 2. Time deposits with the banks 3. Savings deposits with the banks 4. Currency The correct sequence of these assets in the decreasing order of liquidity is (a) 1-4-3-2 (b) 4-3-2-1 (c) 2-3-1-4 (d) 4-1-3-2 50. In the context of Indian economy, ?Open Market Operations? refers to (a) borrowing by scheduled banks from the RBI (b) lending by commercial banks to industry and trade (c) purchase and sale of government securities by the RBI (d) None of the above Placement Paper UPSC CDS Examination Practice question papers,UPSC,SSC,SCRA,CDS CSIR .....previous years solved question papers with detailed explanations, UPSC CDS examination for Indian Navy, Indian Army,Air force...... aptitude elementary mathematics, statistics question papers UPSC CDS Statistics- Question papers Selection Exam 2012 Objective Questions Statistics Part - I 1. Class size is the difference between the class marks of two (a) adjacent classes (Ans) (b) classes (c) alternate classes (d) None of these Hints & Solutions : Class size is the difference between the class marks of two adjacent classes. 2. In bar chart, the gap between the bars is always (a) same (Ans) (b) zero (c) varies (d) can't say Hints & Solutions : Clearly, same. 3. Histogram is (a) one dimensional (b) two dimensional (Ans) (c) three dimensional (d) None of these Hints & Solutions : Clearly, histogram is a two dimensional figure. 4. Joining the mid-points of the respective tops in histogram, we get (a) Bar chart (b) Pie chart (c) Ogive (d) Frequency polygon (Ans) Hints & Solutions : Frequency polygon. 5. The most stable measure of central tendency is (a) the median (b) the mean (Ans) (c) the mode (d) None of these 6. The measure which takes into account all the data items is (a) mean (Ans) (b) frequency (c) mode (d) median Hints & Solutions : Mean 7. Consider the following statement (i) A bar graph is two dimensional (ii) In a histogram, the height as well as the width of each rectangle matters (iii) In a bar graph, not only height, but also width of each rectangle matters. (iv) In a bar graph, height of each rectangle matters and not its width of these statements (a) (i) alone is true (b) All the true (c) (iii) and (iv) are true (Ans) (d) (ii) and (iv) are true 8. In a bar chart, what does matter is (a) length of y-axis (b) length of x-axis (c) width of the bar (Ans) (d) length of the bar Hints & Solutions : Width of the rectangle only makes it pictorially more attractive. 9. Frequency polygon can be drawn after drawing (a) Ogive (b) Bar chart (c) Histogram (Ans) (d) None of these Hints & Solutions : Frequency polygon can be drawn by joining the mid-points of the respective tops in histogram. 10. Pie-chart is drawn with the help of (a) radius of the circle (b) chords of the circle (c) sectors of the circle (Ans) (d) diameter of the circle Hints & Solutions : Pie-chart is drawn with the help of sectors of the circle the angle of sector depends upon the percentage of the quantity showing. 11. The total expenditure incurred by an industry under different heads is best presented as a (a) Pie diagram (Ans) (b) Histogram (c) Frequency polygon (d) Bar chart 12. Which one of the following is a source of data for primary investigations ? (a) News Papers (b) Magazines (c) Questionnaires (Ans) (d) All of these Hints & Solutions : Primary investigation is done by questionnaires. 13. Pie diagrams used to represent statistical data are (a) one dimensional (Ans) (b) two dimensional (c) three dimensional (d) None of these 14. An ogive is used to determine (a) Mean (b) Median (Ans) (c) GM (d) HM Hints & Solutions : An ogive is used to determine the median. 15. The graph of frequency distribution is (i) Histogram (ii) Frequency curve (iii) Ogive (iv) Frequency polygon (a) (i), (ii) and (iv) (Ans) (b) (ii) and (iii) only (c) (i), (ii) and (iii) (d) All are correct 16. A distribution which exhibits real gaps is called (a) Frequency distribution (b) Cumulative frequency distribution (c) Discrete distribution (Ans) (d) Continuous distribution Hints & Solutions : Discrete distribution exhibits real gaps. Also in this type of distribution exact measurements of the units are clearly shown. 17. The mid value of a class interval is 42. If the class size is 10, then the upper and lower limits of the class are (a) 37.5 and 47.5 (b) 47 and 37 (c) 37 and 47 (Ans) (d) 47.5 and 37.5 Hints & Solutions : Let the lower limit be x. Then, the upper limit of class interval = x + 10 ? x + (x + 10) = 42 2 ? 2x + 10 = 84 ? 2x = 74 ? x = 37 ? Lower limit = 37, Upper Limit = 37 + 10 = 47 18. The actual lower class limits of the following classes 10-19, 20-29, 30-39 and 40-49 are (a) 9.5, 19, 29 and 39.5 (b) 10, 20, 30 and 40 (c) 9.5, 19.5, 29.5 and 39.5 (Ans) (d) 18.5, 28.5, 38.5 and 48.5 Hints & Solutions : True lower class limit are obtained by subtracting 0.5 from the lower limit so, clearly 9.5, 19.5, 29.5 and 39.5. 19. State which of the following variable are discrete i. Number of children in a family ii. Wages of workers iii. The ages of students iv. Weights of a set of a students (a) All of these (b) (i), (ii) and (iii) only (Ans) (c) (i) and (ii) only (d) None of these Hints & Solutions : A discrete frequency distribution is such a distribution in which data are presented in a way that exact measurements of the units are clearly shown. So clearly weights of a set of a students is continuous. while other three as discrete. 20. If the mean of five observations x, x + 2, x + 4, x + 6, x + 8 is 11, then the mean of first three observations is : (a) 9 (Ans) (b) 11 (c) 13 (d) None of these Hints & Solutions : Here sum of 5 observations = x + (x + 2) + (x + 4) + (x + 6) + (x + 8) = 5x + 20 ? Mean of total observations = 5x + 20 = 11 5 ? 5x + 20 = 55 ? x = 7 ? Mean of first three = x + (x + 2) + (x + 4) 3 = 3x + 6 = 27 = 9 6 3 UPSC Solved Question Papers UPSC CDS Examination Practice question papers,UPSC,SSC,SCRA,CDS CSIR .....previous years solved question papers with detailed explanations, UPSC CDS examination for Indian Navy, Indian Army,Air force...... aptitude elementary mathematics, statistics question papers UPSC CDS Statistics- Question papers Selection Exam 2012 Objective Questions Statistics 1. The age distribution of workers in a factory is as follows : Age in years No. of workers 20-28 28-36 36-44 44-52 52-60 45 95 100 42 18 If 15% of the total strength starting from lowest age group is retrenched and 20% of the total strength from the highest age groups is given premature retirement, then the age limit of workers retained in the factory is (a) 20-36 (b) 28-44 (Ans) (c) 28-52 (d) 36-52 Hints & Solutions : Total number pf workers = 300 Retrenched = 15% of 300 = 45 These are all from age group 20-28. Premature retired = 20% of 300 = 60 = 18 from age group 52-60 and 42 from age group (44-52) ? Age limit of workers retained is 28-44. 2. The following age groups are included in the proportion indicated Age group Relative proportion in population 12-17 18-23 24-29 30-35 36 + 0.17 0.31 0.27 0.21 0.04 How many of each age-group should be included in a sample of 3000 people to mark the sample representative ? (a) 850, 155, 135, 905, 955 (b) 510, 930, 810, 630, 120 (Ans) (c) 600, 600, 600, 600, 600 (d) 510, 630, 950, 100, 810 3. A data has highest value 120 and lowest value 71. A frequency distribution in descending order with seven classes is to be constructed. The limits of the second class-interval shall be (a) 71 and 78 (b) 78 and 85 (Ans) (c) 113 and 120 (d) 106 and 113 4. If the sum of 11 consecutive natural numbers is 2761, the the middle number is (a) 249 (b) 250 (c) 251 (Ans) (d) 252 Hints & Solutions : According to question x + x + 1 + x + 2 + x + 3 + x + 4 + x + 5 + x + 6 + x + 7 + x + 8 + x + 9 + x + 10 = 2761 ? 11x + 55 = 2761 ? x = 2761 - 55 = 246 11 ? Middle number = x + 5 = 246 + 5 = 251 5. Mean of 100 items is 49. It was discovered that three items which should have been 60, 70, 80 were wrongly read as 40, 20, 50 respectively. The correct mean is (a) 48 (b) 82 (c) 50 (Ans) (d) 80 Hints & Solutions : The sum of items = 49 * 100 - (40 + 20 + 50) + (60 + 70 + 80) = 5000 ? Mean of 100 items = 5000 = 50 100 6. Consider the following statements : 1. In a bar graph not only height but also width of each rectangle matters 2. In a bar graph height of each rectangle matters and not its width 3. In a histogram the height as well as the width of each rectangle matters. 4. A bar graph is two-dimensional of these statements (a) (1) alone is correct (b) (3) alone is correct (c) (2) and (3) are correct (Ans) (d) (1) and (4) are correct 7. A pie chart is to be drawn for representing the following data Items of expenditure Number of families Education Food and clothing House rent Electricity Miscellaneous 150 400 40 250 160 The value of the central angle for food and clothing would be (a) 900 (b) 2080 (c) 1500 (d) 1440 (Ans) Hints & Solutions : Central angle for food and clothing = 400 * 360 = 1440 1000 8. The frequency distribution of some given numbers is Value Frequency 1 2 3 4 5 4 6 f If the mean is known to be 3, then the value of f is (a) 3 (b) 7 (c) 10 (d) 14 (Ans) Hints & Solutions : x f fx 1 2 3 4 5 4 6 f 5 8 18 4f ?f = 15 + f ?fx = 31 + 4f ? Mean = ?fx /?f = 31 + 4f /15 + f ? 3 = 31 + 4f /15 + f ? 45 - 31 = f ? f = 14 9. The average of the squares of the numbers 0, 1, 2, 3, 4, ......, n is (a) 1/2 n (n + 1) (b) 1/6 n (2n + 1) (Ans) (c) 1/6 n (n + 1)(2n + 1) (d) 1/6 n (n + 1) Hints & Solutions : Mean = 12 + 22 + 32 + ............ + n2/ (n + 1) = n (n + 1)(2n + 1) = 1 n (2n + 1) 6 (n + 1) 6 10. If the average of the numbers 148, 146, 144, 142, ..... in AP be 125, then the total numbers in the series will be (a) 18 (b) 24 (Ans) (c) 30 (d) 48 Hints & Solutions : a =148, d = (146 - 148) = -2 Sn = n/2 [2a + (n - 1)d] = 125n ? 125n = n/2 [2 * 148 + (n - 1) * (-2)] ? n2 - 24n = 0 ? n (n - 24) = 0 ? n = 24 [n?0] 11. If the values 1, 1/2, 1/3, 1/4, 1/5, .........., 1/n occur at frequencies 1, 2, 3, 4, 5, ......, n respectively, in a frequency distribution, then the mean is (a) 1 (b) n (c) 1/n (d) 2/n + 1 (Ans) 12. The mean of the values of 1,2,3, ....., n with respectively frequencies x, 2x, 3x, ..... nx is (a) n/2 (b) 1/3(2n + 1) (Ans) (c) 1/6(2n + 1) (d) n/2 Hints & Solutions : ?f = x + 2x + 3x + ..... + nx = x (1 + 2 + 3 + .... + n) = 1/2 nx (n + 1) ?(f * d) = 1 * x + 2 * 2x + 3 * 3x + ...... + n * nx = x(12 + 22 + 32 + .... + n2) = x 1/6 n (n + 1)(2n + 1) Mean = ?(f * d)/?f = 1/3(2n + 1) 13. The arithmetic mean of data with observations a, a + d, a + 2d, ...., a + 2md is (a) a + md (Ans) (b) a + (m - 1)d (c) a + 1/2 md (d) a + 1/2 (m + 1)d Hints & Solutions : S = a + (a + d) + (a + 2d).... + (a + 2md) = 2m + 1 [2a + 2md] 2 = (2m + 1) (a + md) = (a + md) (2m + 1) 14. In a class of 50 students, 10 have failed and their average marks are 28. The total marks obtained by the entire class are 2800. The average marks of those who have passed are (a) 43 (b) 53 (c) 63 (Ans) (d) 70 Hints & Solutions : Total marks of 10 failed students = 28 * 10 = 280 Total marks of 50 students = 2800 ? Total of 40 passed students = 2800 - 280 = 2520 ? Average marks of 40 passed students = 2520/40 = 63 15. Consider the frequency distribution given below Class-interval Frequency 0-10 10-20 20-30 30-40 40-50 4 6 10 16 14 The mean of the above data is (a) 25 (b) 35 (c) 30 (d) 31 (Ans) Hints & Solutions : CI x f fx 0-10 10-20 20-30 30-40 40-50 5 15 25 35 45 4 6 10 16 14 20 90 250 560 630 ?f = 50 ?fx = 1550 ? Mean = ?fx/?f = 1550/50 = 31 16. The mean of 30 given numbers, when it is given that the mean of 10 of them is 12 and the mean of the remaining 20 is 9, is equal to (a) 11 (b) 10 (Ans) (c) 9 (d) 5 _ _ Hints & Solutions : Given that, n1 = 10, x1= 12, n2 = 20, x2 = 9 UPSC NDA General Ability Solved Question Papers NDA National Defence Academy Examination General Intelligence Part A April 2011 Original Question Paper with answers and detailed explanations 1. Complete the series : 11, 12, 17, 18, 23, 24 ... (1) 30 (2) 35 (3) 12 (4) 14 (5) None of these Ans : (5) Explanation : Alternate numbers increase by 6. 2. Complete the series : 1, 8, 27, 64, 125, 216 ..... (1) 343 (2) 512 (3) 729 (4) 1000 (5) None of these Ans : (1) Explanation : The series is 13, 23, 33, 43 ....... Hence missing figure : 73 = 343 3. Suppose you are in charge of a Military Command guarding a bridge and you receive a message from the bridge that the enemy forces have almost reached the other side of the bridge. What telephone message would you send ? (1) Order air-shelling (2) Break the bridge (3) Send more troops (4) Surrender to them (5) None of these Ans : (2) 4. Supply the missing figure : 3, 11, 8, 16, 13 .... 18 (1) 15 (2) 17 (3) 14 (4) 21 (5) None of these Ans : (4) Explanation : In the series take alternate terms and see, it is increasing by 5. Thus missing figure is 16 + 5 = 21 11, 16, (16 + 5), ..... 5. Usha is twice as old as Rita. Three years ago she was three times as old as Rita. How old is Usha now ? (1) 7 years (2) 9 years (3) 6 years (4) 12 years (5) None of these Ans : (4) Explanation : Now U = 2R (U - Usha's age R - Rita's age) Three years ago, (U - 3) = 3(R - 3) ? (2R - 3) = 3(R - 3) (? U = 2R)